Unit 3: The Industrial Revolution and Imperialism

Ace your homework & exams now with Quizwiz!

Company Rule

Belgians, all companies had free access to supplies, no African rights (Leopold II).

What impact did the Taiping Rebellion have on China?

Natural disasters of 1840-1850's. No government help. - Qing army severely weakened due to decrease in enthusiasm and energy. - Qing allow Western influence in the military. - Administrative powers distributed among local authorities. - Led to local militias, they get paid twice the amount the government soldiers do. - 1848 10 million silver taels paid for opium, exchange rate effected negatively and incomes reduced. - China was importing greater than exporting. - Farmers paid 50% of land yield. - Corrupt officials. The rebellion began under the leadership of Hong Xiuquan (1814-64), a disappointed civil service examination candidate who, influenced by Christian teachings, had a series of visions and believed himself to be the son of God, the younger brother of Jesus Christ, sent to reform China. A friend of Hong's, Feng Yunshan, utilized Hong's ideas to organize a new religious group, the God Worshippers' Society (Bai Shangdi Hui), which he formed among the impoverished peasants of Guangxi province. In 1847 Hong joined Feng and the God Worshippers, and three years later he led them in rebellion. On January 1, 1851, he proclaimed his new dynasty, the Taiping Tianguo ("Heavenly Kingdom of Great Peace"), and assumed the title of Tianwang, or "Heavenly King." Their credo—to share property in common—attracted many famine-stricken peasants, workers, and miners, as did their propaganda against the foreign Manchu rulers of China. Taiping ranks swelled, and they increased from a ragged band of several thousand to more than one million totally disciplined and fanatically zealous soldiers, organized into separate men's and women's divisions. Sweeping north through the fertile valley of the Yangtze River (Chang Jiang), they reached the great eastern city of Nanjing. After capturing the city on March 10, 1853, the Taipings halted. They renamed the city Tianjing ("Heavenly Capital") and dispatched a northern expedition to capture the Qing capital at Beijing. That failed, but another expedition into the upper Yangtze valley scored many victories. Taiping Rebellion, radical political and religious upheaval that was probably the most important event in China in the 19th century. It lasted for some 14 years (1850-64), ravaged 17 provinces, took an estimated 20 million lives, and irrevocably altered the Qing dynasty (1644-1911/12).

How did people in the Congo resist? How did the Belgians respond?

Often African nations are described as unstable. There is a great deal of truth in this statement as almost all post-colonial African nations have experienced political violence and severe economic mismanagement during the mid and late twentieth century. Of all African nations, the Democratic Republic of Congo (DRC) has had an especially tumultuous post-colonial experience. Formerly a Belgian colony, the DRC still faces violence in the eastern portion of the country as well as political, economic, and social instability throughout. There are many historical reasons for the DRC's instability but Belgian colonial education policies are a key cause of this instability. In 1884-1885, the Berlin West Africa Conference effectively divided up the African continent amongst the Great Powers of Europe. Attended by the colonial powers of Great Britain, France, Spain, Portugal, Germany, and Belgium, among others, the Conference created artificial state boundaries as well as a colonial system that was in effect for the next sixty years. Among these territories, the Congo was a unique case. Granted to King Leopold II of Belgium, the Congo was a "personal" concession for the King, rather than a colony. The King, not the Belgian government, effectively owned and controlled the Congo. Leopold administered the Congo in a notoriously brutal manner, using it to augment his own personal wealth. The Congo's wealth, which included its numerous rubber trees, was brutally extracted using what was basically slave labor. This rubber was then exported to fuel the industrial growth of both nineteenth- and early twentieth-century Europe and America. King Leopold II of Belgium, as a Garter Knight. Despite his growing reliance on the wealth of the Congo, Leopold never visited this territory himself. By 1908, the territory was so poorly managed that an international furor condemning Leopold had erupted. That same year, in an attempt to stem this furor, the Congo was ceded to Belgium and placed under the control of the Belgian government, not its king. Belgium then administered the Congo as a colony until independence in 1960. Unlike other early twentieth-century colonial powers in Africa, Belgium did not directly oversee the education of the Congo's indigenous population. Rather, it turned the responsibility for education over to missionaries. In 1908, the Congo had 587 missionaries, mostly Catholic, who educated only 46,075 students, a very small fraction of the many residents. This small number of students stemmed from many factors. The number of missionaries was insufficient to educate a large population. But the missionary's educational agenda, which often undermined indigenous African culture and promoted colonial domination, also deterred many Africans from pursuing European educations. Throughout the late nineteenth and early twentieth centuries, the primary role of the Church, both in religion and education, was to promote colonialism. Many Congolese villagers at first avoided the mission schools because they had a religious agenda that threatened to undermine their cultural values and beliefs. The Belgian Congo is often cited as one of the most brutal and exploitative colonial regimes in modern history. It stands as an extreme example of the cruelty of European rule in Africa for the sake of economic gain. King Leopold II and the Congo Free State In the last decades of the 19th century, the largely uncharted African continent was overrun by a sudden wave of European imperialist expansion. Driven by nationalist pride, imperial ambitions, and the hope of acquiring vast new economic resources, the leading nations of Europe each sought to claim a piece of the African 'cake' for themselves. The Berlin Conference of 1884 through 1885 formalized many of the major powers' claims in Africa and granted the coveted Congo River basin to King Leopold II of Belgium. Initially called the Congo Free State, the colony remained a personal possession of King Leopold II from 1885 until 1908 when it was taken over by the Belgian government and renamed the Belgian Congo. But, why all the bother over Africa? Most African colonies at the time, including the Congo Free State, were created primarily for the economic exploitation of natural resources and labor. The major exports from the Congo region included ivory, rubber, and precious minerals, all of which were highly profitable and in great demand in Europe. To maximize the profitability, the interior regions of the Congo that could not be reached by river and steamship were steadily opened up through the construction of railroads. Slavery, Genocide, and Economic Exploitation The Congo Free State as it existed under Leopold II is largely known to history for its brutal exploitation of the native Congolese population and the mass death that resulted. Under Leopold II there were virtually no laws or restrictions protecting the native Congolese and their lands. The peoples of the Congo River basin were forced to work as porters, miners, rubber-tappers, woodcutters, and railway builders for European interests. Because there was little oversight and no form of organized government control, Europeans were free to adopt brutal policies of kidnapping, mutilation, robbery, and murder to extract desired labor and resources from the local population. The harvesting of rubber was a particularly arduous task. Rubber was in great demand in Europe for use in the manufacture of bicycle and automobile tires. But, how could Europeans with limited resources and manpower force large numbers of local peoples to harvest rubber deep in the jungles of Africa? A colonial army called the force publique (public force) was created largely from local Africans and a handful of Belgian officers in order to marshal labor, quell revolts, and enforce the collection of rubber and ivory. A common tactic used in the Congo Free State was to demand a certain quota of rubber from each village. Missing a quota was punished violently. It was not uncommon for the hands and feet of men, women, and children to be amputated as punishment for not collecting enough rubber or ivory. Hostages were taken from villages and used as leverage when demanding quotas. Villagers knew that not gathering the specified quota of goods could mean the execution of their family. Such inhumane policies drove many villagers of the Congo to revolt and resist colonial rule. Rebellions were put down swiftly and violently, often by killing all those who refused to work. The bodies of rebels were often displayed as a warning to others. While the death toll in the Congo Free State can never be truly known due to a lack of accurate records, historians have offered estimates as high as ten million dead between 1885 and 1908. Colonial mismanagement and oppression led not only to the killing and maiming of native peoples, but also to overwork, disease, starvation, and a host of other factors that all combined in a massive loss of life.

What are labor unions, and why did they pose a challenge to capitalism? What reforms did they lead to?

An organized association of workers, often in a trade or profession, formed to protect and further their rights and interests. Employers argued that these were dangerous to business and the country. Labor Unions affected the Industrial Revolution because the Unions could either support a candidate or dislike them, which seemed to have a large effect on their popularity and success in politics. Labor Unions affected the Industrial Revolution because the Unions could either support a candidate or dislike them, which seemed to have a large effect on their popularity and success in politics. In general terms, business owners have answered. Labor is a commodity. It's something that a worker can sell for whatever the market will bear. The price of labor—that is, wages—will be dictated by supply and demand. The hours and conditions of work will be set by the employer or reached by some agreement between the employer and individual workers. That is the employer's right. Workers have generally offered a different definition. Labor is an integral part of a person's life. Employees might recognize that supply and demand will influence wages, but they feel that labor has a special moral dimension that makes it different than a simple commodity. After all, a person's labor can't be separated from his or her life. Workers have rights on the job and rights to decent wages. The ideological dispute has also involved individual versus collective rights. Employers pushed individual rights. They insisted on their right to deal with any individual worker. Unions pushed for group rights. In collective bargaining, a critical concept in labor history, the union came to an agreement with the employer and that agreement then became binding for all. A shop clerk in the early-19th century noted that a printer "often came into our store for a pitcher of ale to cheer up the boys in the printing office nearby. It wasn't at all unusual in those days for a boss to drink with his employees on the job, or for a worker to take a day off when he felt like it. By the second half of the century, all that had changed, but the memory of a different labor situation survived. Workers in the big industrial factories knew that their grandfathers had enjoyed a looser, more relaxed climate at work, but the conditions that came with the big factories were shockingly different. Workers had to be disciplined in order to keep the enterprise operating efficiently. No more drinking on the job. No more setting their own hours or pace of work. A say in how the work was done, a traditional prerogative of craftsmen, was reserved solely for the boss. Regimentation wasn't easy for Americans to swallow, but business owners considered it the key to productivity and profit in the industrial age. Working conditions were often brutal and dangerous. Mines caved in, workers breathed toxic dust or were mangled in machinery. Factory workers were rigidly controlled and often fined for making the least mistake. Even bathroom breaks required the boss' permission. Outside the workplace, frequent financial panics left millions unemployed. In the face of all these problems, workers increasingly began to organize. Many skilled workers had already formed craft unions, which predated the Industrial Revolution. But faced with the rapidly growing scale of industry and power of employers after the Civil War, many workers felt they needed an organization that was national in scope and big enough to stand up to Big Business.

Opium War

Between China and British because of the devastation opium had caused China. China wanted British out but British refused to leave. 1840: Beginning of Modern China. Fought between the British and Qing China beginning in 1839; fought to protect British trade in opium; resulted in resounding British victory, opening of Hong Kong as British port of trade. British attempted to offset trade imbalance for Chinese tea by shipping highly addictive opium from Brit-ruled India to China in exchange for silver even though opium was outlawed in China. The war was started by the British against the Chinese for their efforts to force the halt of opium sales in China. China is easily defeated. (Second) British fought China to re-negotiate Treaty of Nanjing.

What effect did colonization have on Africa?

Between the 1870s and 1900, Africa faced European imperialist aggression, diplomatic pressures, military invasions, and eventual conquest and colonization. At the same time, African societies put up various forms of resistance against the attempt to colonize their countries and impose foreign domination. By the early twentieth century, however, much of Africa, except Ethiopia and Liberia, had been colonized by European powers. The European imperialist push into Africa was motivated by three main factors, economic, political, and social. It developed in the nineteenth century following the collapse of the profitability of the slave trade, its abolition and suppression, as well as the expansion of the European capitalist Industrial Revolution. The imperatives of capitalist industrialization—including the demand for assured sources of raw materials, the search for guaranteed markets and profitable investment outlets—spurred the European scramble and the partition and eventual conquest of Africa. Thus the primary motivation for European intrusion was economic. But other factors played an important role in the process. The political impetus derived from the impact of inter-European power struggles and competition for preeminence. Britain, France, Germany, Belgium, Italy, Portugal, and Spain were competing for power within European power politics. One way to demonstrate national preeminence was through the acquisition of territories around the world, including Africa. The social factor was the third major element. As a result of industrialization, major social problems grew in Europe: unemployment, poverty, homelessness, social displacement from rural areas, and so on. These social problems developed partly because not all people could be absorbed by the new capitalist industries. One way to resolve this problem was to acquire colonies and export this "surplus population." This led to the establishment of settler-colonies in Algeria, Tunisia, South Africa, Namibia, Angola, Mozambique, and central African areas like Zimbabwe and Zambia. Eventually the overriding economic factors led to the colonization of other parts of Africa. Thus it was the interplay of these economic, political, and social factors and forces that led to the scramble for Africa and the frenzied attempts by European commercial, military, and political agents to declare and establish a stake in different parts of the continent through inter-imperialist commercial competition, the declaration of exclusive claims to particular territories for trade, the imposition of tariffs against other European traders, and claims to exclusive control of waterways and commercial routes in different parts of Africa. This scramble was so intense that there were fears that it could lead to inter-imperialist conflicts and even wars. To prevent this, the German chancellor Otto von Bismarck convened a diplomatic summit of European powers in the late nineteenth century. This was the famous Berlin West African conference (more generally known as the Berlin Conference), held from November 1884 to February 1885. The conference produced a treaty known as the Berlin Act, with provisions to guide the conduct of the European inter-imperialist competition in Africa.

Which reasons did the European empires have when they took over other countries?

European countries began exploring and seeking to dominate the rest of the world during the 15th and 16th centuries, thanks to their ability to control sea routes and to the discovery of the American continent. In the 19th century, energized by the industrial revolution and under pressure from a rapidly growing population, Europe launched a new period of colonial expansion, inspired by the discovery of new markets, new areas for the settlement of Europe's poor migrants, and the desire to " civilize the barbarian nations ". Marxist scholars would see imperialism as a relationship involving economic exploitation, with economic benefits flowing from subject to ruler; some conceive of this in the broadest terms, with imperialism represented by an unequal economic relationship that involves a degree of continuing dependency going well beyond formal colonial rule. Critics of these economic models, however, would see imperialism in political terms and restrict its application to the assertion of political or strategic goals by one society over another. For the purposes of this entry, imperialism is defined to involve the subordination of one society by another to the benefit of the latter, in a relationship that involves some degree of control over a period of time. In historical terms, imperialism is herein taken to refer to the subjugation of the states and societies of Africa, Asia, and the Pacific by the European powers during the nineteenth and twentieth centuries. Such expansion, of course, did not begin in this period and can be traced back to the Iberian voyages of discovery of the fifteenth century. Yet clearly this phenomenon took on new impetus in the nineteenth century, when much of Africa, Asia, and the Pacific came under European rule in what is sometimes called the "New Imperialism." What characterized this period was how extensive this process became, not only in incorporating most of the world under European rule but also in involving most of the major European powers. By 1900, the British ruled 400 million subjects and a quarter of the globe, while French possessions encompassed 6 million square miles and 52 million subjects. Germany, united only in 1871, by 1900 had acquired an empire of 1 million square miles and 15 million people, while Italy and Belgium also obtained significant overseas territories in the years before 1914. By that year most of the globe—with China and the Middle East the main exceptions—had experienced formal European rule; this was to expand to include the Middle East after 1919 and was to continue in most parts of the world until after 1945. This was a remarkable transformation in the relationship between Europe and the wider world. Few subjects have generated as much controversy as that of European imperialism in the nineteenth and twentieth centuries. Because of the term's pejorative connotations, even attempting to define it causes difficulties. What, precisely, constitutes an empire? Is any relationship between two societies involving an imbalance of power in some way "imperial"? For many years, imperialism was narrowly defined as applying only to actual conquest and administration of one state by another. More recently, it has been accepted that imperialism can be perceived in broader terms, and that a variety of methods of exerting influence by one state over another, stopping short of conquest, can amount to imperialism; this "informal" imperialism, it is argued, is as imperial as the more "formal" conquest more usually described. Even accepting this distinction, there is considerable debate over the meaning of the term.

Capitalists

Industrial Revolution. The productivity gains of capitalist production began a sustained and unprecedented increase at the turn of the 19th century, in a process commonly referred to as the Industrial Revolution.

Treaty of Nanjing

The Treaty of Nanking or Nanjing was a peace treaty which ended the First Opium War (1839-42) between the United Kingdom and the Qing dynasty of China on 29 August 1842. It was the first of what the Chinese later called the unequal treaties on the ground that Britain had no obligations in return. Treaty of Nanjing was the result of China's humiliating defeat at the hands of the British in the Opium War. Britain took the island of Hong Kong, a very important trading port.

Reasons motivating European Imperialism

Various motives prompt empires to seek to expand their rule over other countries or territories. These include economic, exploratory, ethnocentric, political, and religious motives. Economic: Imperial governments, and/or private companies under those governments, sought ways to maximize profits. Economic expansion demanded cheap labor, access to or control of markets to sell or buy products, and natural resources such as precious metals and land; governments have met these demands by hook (tribute) or by crook (plunder). After the advent of the Industrial Revolution, dependent colonies often provided to European factories and markets the raw materials they needed to manufacture products. Imperial merchants often established trading posts and warehouses, created transportation infrastructure, and sought control over strategic choke points, such as the Suez Canal in Egypt (which allows boats to cut thousands of miles of travel time between Asia and Europe). Imperial powers often competed with each over for the best potential resources, markets, and trade. Exploratory: Imperial nations or their citizens wanted to explore territory that was, to them, unknown. Sometimes they did this for the purpose of medical or scientific research. At other times, they did it for the sense of adventure. Invariably, imperial explorers sought to discover, map, and claim territory before their imperial competition did, partly for national and personal glory and partly to serve the imperialist goal of expansion. Ethnocentric: Imperial nations sometimes believed that their cultural values or beliefs were superior to other nations or groups. Imperial conquest, they believed, would bring successful culture to inferior people. In the late 19th century, for example, European powers clung to the racist belief that inferior races should be conquered in order to "civilize" them. The Europeans acted on their ethnocentrism, the belief that one race or nation is superior to others. Political: Patriotism and growing imperial power spurred countries to compete with others for supremacy. It's a matter of national pride, prestige and security. Empires sought strategic territory to ensure access for their navies and armies around the world. The empire must be defended and, better yet, expanded. Political motives were often triggered as responses to perceived threats to the security or prestige of the imperial power or its citizens abroad. Religious: During imperial expansion, religious people sometimes set out to convert new members of their religion and, thus, their empire. Christian missionaries from Europe, for example, established churches in conquered territories during the nineteenth century. In doing so, they also spread Western cultural values. Typically, missionaries spread the imperial nation's language through educational and religious interactions, although some missionaries helped to preserve indigenous languages. British missionaries led the charge to stop the slave trade in the nineteenth century, while others, such as French missionaries in Vietnam during the same time period, clamored for their country to take over a nation.

The connections between the Industrial Revolution and European Imperialism

Imperialism in Asia traces its roots back to the late fifteenth century with a series of voyages that sought a sea passage to India in the hope of establishing direct trade between Europe and Asia in spices. Before 1500 European economies were largely self-sufficient, only supplemented by minor trade with Asia and Africa. Within the next century, however, European and Asian economies were slowly becoming integrated through the rise of new global trade routes; and the early thrust of European political power, commerce, and culture in Asia gave rise to a growing trade in lucrative commodities—a key development in the rise of today's modern world free market economy. In the sixteenth century, the Portuguese established a monopoly over trade between Asia and Europe by managing to prevent rival powers from using the water routes between Europe and the Indian Ocean. However, with the rise of the rival Dutch East India Company, Portuguese influence in Asia was gradually eclipsed. Dutch forces first established independent bases in the East (most significantly Batavia, the heavily fortified headquarters of the Dutch East India Company) and then between 1640 and 1660 wrestled Malacca, Ceylon, some southern Indian ports, and the lucrative Japan trade from the Portuguese. Later, the English and the French established settlements in India and established a trade with China and their own acquisitions would gradually surpass those of the Dutch. Following the end of the Seven Years' War in 1763, the British eliminated French influence in India and established the British East India Company as the most important political force on the Indian Subcontinent. Before the Industrial Revolution in the mid-to-late nineteenth century, demand for oriental goods remained the driving force behind European imperialism, and (with the important exception of British East India Company rule in India) the European stake in Asia remained confined largely to trading stations and strategic outposts necessary to protect trade. Industrialisation, however, dramatically increased European demand for Asian raw materials; and the severe Long Depression of the 1870s provoked a scramble for new markets for European industrial products and financial services in Africa, the Americas, Eastern Europe, and especially in Asia. This scramble coincided with a new era in global colonial expansion known as "the New Imperialism," which saw a shift in focus from trade and indirect rule to formal colonial control of vast overseas territories ruled as political extensions of their mother countries. Between the 1870s and the beginning of World War I in 1914, the United Kingdom, France, and the Netherlands — the established colonial powers in Asia — added to their empires vast expanses of territory in the Middle East, the Indian Subcontinent, and South East Asia. In the same period, the Empire of Japan, following the Meiji Restoration; the German Empire, following the end of the Franco-Prussian War in 1871; Tsarist Russia; and the United States, following the Spanish-American War in 1898, quickly emerged as new imperial powers in East Asia and in the Pacific Ocean area.

Why did both the British and Chinese think they were right about the "Opium War?"

In 1839, England went to war with China because it was upset that Chinese officials had shut down its drug trafficking racket and confiscated its dope. Stating the historical record so plainly is shocking — but it's true, and the consequences of that act are still being felt today. The Qing Dynasty, founded by Manchurian clans in 1644, expanded China's borders to their farthest reach, conquering Tibet, Taiwan and the Uighur Empire. However, the Qing then turned inward and isolationist, refusing to accept Western ambassadors because they were unwilling to proclaim the Qing Dynasty as supreme above their own heads of state. Foreigners — even on trade ships — were prohibited entry into Chinese territory. The exception to the rule was in Canton, the southeastern region centered on modern-day Guangdong Province, which adjoins Hong Kong and Macao. Foreigners were allowed to trade in the Thirteen Factories district in the city of Guangzhou, with payments made exclusively in silver. The British gave the East India Company a monopoly on trade with China, and soon ships based in colonial India were vigorously exchanging silver for tea and porcelain. But the British had a limited supply of silver. Opium War: Starting in in the mid-1700s, the British began trading opium grown in India in exchange for silver from Chinese merchants. Opium — an addictive drug that today is refined into heroin — was illegal in England, but was used in Chinese traditional medicine. However, recreational use was illegal and not widespread. That changed as the British began shipping in tons of the drug using a combination of commercial loopholes and outright smuggling to get around the ban. Chinese officials taking their own cut abetted the practice. American ships carrying Turkish-grown opium joined in the narcotics bonanza in the early 1800s. Consumption of opium in China skyrocketed, as did profits. The Daoguang Emperor became alarmed by the millions of drug addicts — and the flow of silver leaving China. As is often the case, the actions of a stubborn idealist brought the conflict to a head. In 1839 the newly appointed Imperial Commissioner Lin Zexu instituted laws banning opium throughout China. He arrested 1,700 dealers, and seized the crates of the drug already in Chinese harbors and even on ships at sea. He then had them all destroyed. That amounted to 2.6 million pounds of opium thrown into the ocean. Lin even wrote a poem apologizing to the sea gods for the pollution. Angry British traders got the British government to promise compensation for the lost drugs, but the treasury couldn't afford it. War would resolve the debt. But the first shots were fired when the Chinese objected to the British attacking one of their own merchant ships. Chinese authorities had indicated they would allow trade to resume in non-opium goods. Lin Zexu even sent a letter to Queen Victoria pointing out that as England had a ban on the opium trade, they were justified in instituting one too. It never reached her, but eventually did appear in the Sunday Times. Instead, the Royal Navy established a blockade around Pearl Bay to protest the restriction of free trade ... in drugs. Two British ships carrying cotton sought to run the blockade in November 1839. When the Royal Navy fired a warning shot at the second, The Royal Saxon, the Chinese sent a squadron of war junks and fire-rafts to escort the merchant. HMS Volage's Captain, unwilling to tolerate the Chinese "intimidation," fired a broadside at the Chinese ships. HMS Hyacinth joined in. One of the Chinese ships exploded and three more were sunk. Their return fire wounded one British sailor. Seven months later, a full-scale expeditionary force of 44 British ships launched an invasion of Canton. The British had steam ships, heavy cannon, Congreve rockets and infantry equipped with rifles capable of accurate long range fire. Chinese state troops — "bannermen" — were still equipped with matchlocks accurate only up to 50 yards and a rate of fire of one round per minute. Antiquated Chinese warships were swiftly destroyed by the Royal Navy. British ships sailed up the Zhujiang and Yangtze rivers, occupying Shanghai along the way and seizing tax-collection barges, strangling the Qing government's finances. Chinese armies suffered defeat after defeat. When the Qing sued for peace in 1842, the British could set their own terms. The Treaty of Nanjing stipulated that Hong Kong would become a British territory, and that China would be forced to establish five treaty ports in which British traders could trade anything they wanted with anybody they wanted to. A later treaty forced the Chinese to formally recognize the British as equals and grant their traders favored status. More War, More Opium: Imperialism was on the upswing by the mid-1800s. France muscled into the treaty port business as well in 1843. The British soon wanted even more concessions from China — unrestricted trade at any port, embassies in Beijing and an end to bans on selling opium in the Chinese mainland. One tactic the British used to further their influence was registering the ships of Chinese traders they dealt with as British ships. The pretext for the second Opium War is comical in its absurdity. In October 1856, Chinese authorities seized a former pirate ship, the Arrow, with a Chinese crew and with an expired British registration. The captain told British authorities that the Chinese police had taken down the flag of a British ship.

What led to the Sepoy Rebellion?

Britain had land on almost every continent, and on the guns there is a greased cartridge with pig and cow fat which goes against the Hindu and Muslim religious beliefs -the Sepoys to refuse to bite it off which leads to their arrests -this threw the EIC into crisis

Industrialization

Cause & Effect of Industrialization: - population increase - transportation improvements (railroads; new markets) - inventions: boost production, new forms of energy, farming innovations - investment internally + externally; capital - favorable government policies (laissez faire; LLC) - natural resources -shifted the world balance of power -widened gap between industrialized and non-industrialized countries while strengthening economic ties -industrialized countries viewed poor countries as markets for their manufactured goods ==> large equality -European countries began seizing colonies for economic resources -imperialism gave more wealth; imperialism born The process of developing machine production of goods. The development of industries for the machine production of goods. Explain the spread of industrialization in Europe: -European merchants started to adopt to Britain's profitable new methods of manufacturing goods BELGIUM: rich deposits of iron and coal and waterways; they got secrets for building spinning machinery machinery, steam engines, and railway locomotives GERMANY: pockets of industrialization appeared in coal areas; copied British model and imported equipment and engineers; children sent to England to learn industry; built railroads that linked cities; economic strength spurred ability as military power EUROPE: agricultural pocketed cities still had pockets or industrialization; continual growth led to railroads

Tokugawa Shogunate

Military rulers of japan who successfully united japan politically by the early 17th c. and established a "closed door" policy toward european encroachments. 1450-1750 : The military rulers of Japan who successfully unified Japan by the early seventeenth century and established a "closed door" policy toward European encroachments. They largely closed their country off from the emerging world of European commerce, although they maintained their trading ties to China and Korea.

Water frame

Richard Arkwright: English inventor and entrepreneur who became the wealthiest and most successful textile manufacturer of the early Industrial Revolution. He invented the water frame, a machine that, with minimal human supervision, could spin several threads at once. 1780's; Richard Arkwright; powered by horse or water; turned out yarn much faster than cottage spinning wheels, led to development of mechanized looms. A device invented by Richard Arkwright in 1796 which used water power from running streams to drive spinning wheels. Spinning frame when water power was used to drive it; invented by Richard Arkwright; reduced the amount of human labor needed to increase spindle count.

How did the Industrial Revolution make European imperialism possible? Ex: Europeans were certainly interested in controlling Africa before the 1800s. Why did they conquer Africa only in the 1800s?

The Industrial Revolution lead to imperialism because it gave the motive and the ability to imperialize Asia and Africa. The first major inventions that began the Industrial Revolution were the steam engine and spinning jenny. The steam engine was created in the early 1700s, and perfected in the late 1700s. New Imperialism. With the wealth of the Industrial Revolution burning in their pockets, the powerful nations of Europe were ready to formally expand their empires into Asia and Africa. ... In this period of New Imperialism, Europeans began to seek formal political control over foreign and overseas areas. Between the 1870s and 1900, Africa faced European imperialist aggression, diplomatic pressures, military invasions, and eventual conquest and colonization. At the same time, African societies put up various forms of resistance against the attempt to colonize their countries and impose foreign domination. By the early twentieth century, however, much of Africa, except Ethiopia and Liberia, had been colonized by European powers. The European imperialist push into Africa was motivated by three main factors, economic, political, and social. It developed in the nineteenth century following the collapse of the profitability of the slave trade, its abolition and suppression, as well as the expansion of the European capitalist Industrial Revolution. The imperatives of capitalist industrialization—including the demand for assured sources of raw materials, the search for guaranteed markets and profitable investment outlets—spurred the European scramble and the partition and eventual conquest of Africa. Thus the primary motivation for European intrusion was economic. But other factors played an important role in the process. The political impetus derived from the impact of inter-European power struggles and competition for preeminence. Britain, France, Germany, Belgium, Italy, Portugal, and Spain were competing for power within European power politics. One way to demonstrate national preeminence was through the acquisition of territories around the world, including Africa. The social factor was the third major element. As a result of industrialization, major social problems grew in Europe: unemployment, poverty, homelessness, social displacement from rural areas, and so on. These social problems developed partly because not all people could be absorbed by the new capitalist industries. One way to resolve this problem was to acquire colonies and export this "surplus population." This led to the establishment of settler-colonies in Algeria, Tunisia, South Africa, Namibia, Angola, Mozambique, and central African areas like Zimbabwe and Zambia. Eventually the overriding economic factors led to the colonization of other parts of Africa. Thus it was the interplay of these economic, political, and social factors and forces that led to the scramble for Africa and the frenzied attempts by European commercial, military, and political agents to declare and establish a stake in different parts of the continent through inter-imperialist commercial competition, the declaration of exclusive claims to particular territories for trade, the imposition of tariffs against other European traders, and claims to exclusive control of waterways and commercial routes in different parts of Africa.

Why did Japan industrialize and not China?

The Meiji Restoration (Meiji Ishin), also known as the Meiji Ishin, Renovation, Revolution, Reform, or Renewal, was an event that restored practical imperial rule to Japan in 1868 under Emperor Meiji. They learned about Western society like military and economics. They used certain aspects to modernize Japan and the emperor supported industrialization which helped them become an imperial power. However, when they tried to sell opium in China, the Chinese shut the ports and hired an honest man (important because he didn't accept bribes) to keep them out. Unwilling to lose one of their best customers so easily, Britain, with some help from France, absolutely crushed China in the First Opium War. The undisciplined horde of Chinese soldiers, some of whom still fought with swords, was no match for the modern British guns. The effect of this war was that China opened her ports to Europeans to an extent never before seen. Opium wasn't the only commodity that could now be traded in China, either. Europeans set up trading posts all throughout the country, along the coast and rivers, where they could sell all manner of things. Importantly, though, was the fact that China was now open to Europeans, and not just their goods. Missionaries flooded China, seeking converts after centuries of being locked out. One convert to Protestant Christianity would later establish his own kingdom of followers in southern China, in what would come to be known as the Taiping Rebellion - costing millions of lives. The advantages consisted of: Japan already knew the benefits of imitation; Japan's slower government growth had allowed a stronger, more autonomous merchant tradition even as both societies became more commercial in the 17th and early 18th centuries; Japan maintained political and economic vigor into the 19th century; Feudal traditions, though declining under the Tokugawa Shogunate, also limited the heavy hand of government controls while simulating some sense of competitiveness, as in the West; Japan enjoyed increasing industrial success and had a conservative state that would yield after World War II to a more fully parliamentary form; Japan's industrial lead remains, but China's economy is stirring.

Reasons for Imperialism

Economic: Imperial governments, and/or private companies under those governments, sought ways to maximize profits. Economic expansion demanded cheap labor, access to or control of markets to sell or buy products, and natural resources such as precious metals and land; governments have met these demands by hook (tribute) or by crook (plunder). After the advent of the Industrial Revolution, dependent colonies often provided to European factories and markets the raw materials they needed to manufacture products. Imperial merchants often established trading posts and warehouses, created transportation infrastructure, and sought control over strategic choke points, such as the Suez Canal in Egypt (which allows boats to cut thousands of miles of travel time between Asia and Europe). Imperial powers often competed with each over for the best potential resources, markets, and trade. Exploratory: Imperial nations or their citizens wanted to explore territory that was, to them, unknown. Sometimes they did this for the purpose of medical or scientific research. At other times, they did it for the sense of adventure. Invariably, imperial explorers sought to discover, map, and claim territory before their imperial competition did, partly for national and personal glory and partly to serve the imperialist goal of expansion. Ethnocentric: Imperial nations sometimes believed that their cultural values or beliefs were superior to other nations or groups. Imperial conquest, they believed, would bring successful culture to inferior people. In the late 19th century, for example, European powers clung to the racist belief that inferior races should be conquered in order to "civilize" them. The Europeans acted on their ethnocentrism, the belief that one race or nation is superior to others. Political: Patriotism and growing imperial power spurred countries to compete with others for supremacy. It's a matter of national pride, prestige and security. Empires sought strategic territory to ensure access for their navies and armies around the world. The empire must be defended and, better yet, expanded. Political motives were often triggered as responses to perceived threats to the security or prestige of the imperial power or its citizens abroad. Religious: During imperial expansion, religious people sometimes set out to convert new members of their religion and, thus, their empire. Christian missionaries from Europe, for example, established churches in conquered territories during the nineteenth century. In doing so, they also spread Western cultural values. Typically, missionaries spread the imperial nation's language through educational and religious interactions, although some missionaries helped to preserve indigenous languages. British missionaries led the charge to stop the slave trade in the nineteenth century, while others, such as French missionaries in Vietnam during the same time period, clamored for their country to take over a nation.

Did British rule help or harm India?

Both, just depends what point of view you have. British rule in the India was worst thing which could happen to any country. The british empire started its empire as pirates, it looted, destroyed and invaded several kingdoms within India and last battle was in 1857 when last mughal emperor was deposed thus putting an end of 1000 years of Muslim rule in India. Benefits: In the Cause of Humanity: Abolition of suttee and infanticide. Destruction of Dacoits, Thugs, Pindarees, and other such pests of Indian society. Allowing remarriage of Hindu widows, and charitable aid in time of famine. Glorious work all this, of which any nation may well be proud, and such as has not fallen to the lot of any people in the history of mankind. In the Cause of Civilization: Education, both male and female. Though yet only partial, an inestimable blessing as far as it has gone, and leading gradually to the destruction of superstition, and many moral and social evils. Resuscitation of India's own noble literature, modified and refined by the enlightenment of the West. Politically: Peace and order. Freedom of speech and liberty of the press. Higher political knowledge and aspirations. Improvement of government in the native states. Security of life and property. Freedom from oppression caused by the caprice or greed of despotic rulers, and from devastation by war. Equal justice between man and man (sometimes vitiated by partiality to Europeans). Services of highly educated administrators, who have achieved the above-mentioned results. Materially: Loans for railways and irrigation. Development of a few valuable products, such as indigo, tea, coffee, silk, etc. Increase of exports. Telegraphs. Generally: A slowly growing desire of late to treat India equitably, and as a country held in trust. Good intentions. No nation on the face of the earth has ever had the opportunity of achieving such a glorious work as this. I hope in the credit side of the account I have done no injustice, and if I have omitted any item which anyone may think of importance, I shall have the greatest pleasure in inserting it. I appreciate, and so do my countrymen, what England has done for India, and I know that it is only in British hands that her regeneration can be accomplished. Now for the debit side. India was the jewel in the crown of the British Empire. As well as spices, jewels and textiles, India had a huge population. Soldiering was an honourable tradition in India and the British capitalised on this. They regimented India's manpower as the backbone of their military power.

What happened in the Belgian Congo? How did it make Europe rich at the expense of the Belgians?

Image result for What happened in the Belgian Congo? Colonial rule in the Congo began in the late 19th century. King Leopold II of Belgium attempted to persuade the Belgian government to support colonial expansion around the then-largely unexplored Congo Basin. Their ambivalence resulted in Leopold's creating a colony on his own account. The loose name used to describe the Congo colony after 1908. It was controlled by a Belgian management organisation that consisted of members of the government, church and powerful mining companies. While conditions in the Congo did improve, it was still predominantly an economic colony. This period of Congolese history ends when the Congo gain their independence in 1960. Prior to the arrival of European people, Africa was divided among the tribes that lived to the best of their ability. These tribes were often hunter-gather's who lived day to day. However in the another part of the world, European Powers began to expand and eventually their gaze turned towards the untouched natural goods that Africa could bring. These powers would seek to spread their power as far out as they could possibly could. It went beyond an act to obtain natural goods but to a power struggle between who would be far more powerful than anyone else. This was a race to build the biggest empire. Beginning in the 1800's European countries met to begin carving up Africa into colonies for Europe in what would become the Scramble for Africa. This was when African's destiny was determined by white men. Their history was also going to be determined by white men who record the many events of the Congo(Benson, Osarhieme). Many Europeans followed a hierarchy of races that saw western Europeans on the top and put Africans at the bottom. This of course was not a biological hierarchy but rather a hierarchy that was created socially through years of racist practices that the Europeans took part in. This Scramble for Africa left African natives in control of European colonialists who took advantage of the raw goods and people of these countries. None of these more apparent than those of the Congo. The Republic of the Congo still struggle with their economy due to its historical inability to form on its own. The same can be said for other countries in Africa as the current African Economies struggle to maintain strong markets due to the large amount of Imperialistic influence both currently and historically. Due to such influence they were never given the opportunity to form on their own, rather they were told how to form and in what way. This leaves post-colonial economies struggling to form while others around it are thriving in full force. The best way to truly understand the issues of the Congo's economy is to first look at the harsh imperialistic rule King Leopold of Belgium imposed the country of the Congo, next I will explore how Leopold went about keeping his colony and what he did to preserve it and finally I will look at the current socioeconomic status of the Congo and how Leopold influenced this. Imperialism- The Longest Reach First in order to truly understand the unique situation that the Congo finds themselves in one must first look at the imperialistic rule that King Leopold imposed on the Congo. Leopold II inherited his throne from his father in the late 1800's and would come to have the longest rule of a Belgian Monarch. In a time when other European countries where expanding into empires and benefiting greatly from these empires, King Leopold II decided that he too would take on a colony of his own. However it wasn't a colony for Belgium, it would be his own colony. The King of Belgium sent in an explorer by the name of Henry Morton Stanley to talk with the chiefs of all the tribes of the Congo. By June of 1884 Stanley had gained the trust of all of the tribes in the Congo as well as had them sign treaties allowing Leopold the Second to begin to use the natural goods that the Congo had(Hoschschild, 61-74). From here, Leopold II mostly targeted the rubber that flourished in the trees of the Congo. Each day he would instruct the natives of the Congo to collect a certain quota of rubber each day and those who did not follow the quota of rubber each day were punished, often times having their hand severed off. The men of the Congo who would refuse to go into the forest to collect rubber had their wives captured and held hostage at gun point until the men went in to the Jungle to collect rubber for the Anglo-Belgian India Rubber Company or A.B.I.R.. Leopold looked down on the people of the Congo so much that he even informed the Christian Missionaries that went to convert natives of the Congo that it was a waste of their time. King Leopold informed them that the Congolese were no only unworthy of learning of God but were also too stupid to comprehend it. The King of Belgium said not to share "more than one cigarette" for they might then try to take advantage of the missionaries(Leopold). Rather the Congolese should be kept on a short leash and only be interacted with when they were being forced to collect rubber or when they were being punished. As one would expect the King of Belgians gained a lot from the exploitation of the Congo. Along with the large sums of money the he gained from the ABIR, Leopold also had certain commodities brought in from the Congo so that people would experience the picturesque Congo that Belgians had in their heads. King Leopold even had "Red mahogany from [the Congo] paneling the bed room of his private railway car..."(Hochschild, 175). Animals from the Congo also came into captivity so that they could be shown to the Belgian people just so that they could see what they thought the Congo was like. Completely disregarding the people of the Congo and the harsh punishment that they now underwent.

Belgian Rule over Congo-Force Publique, etc.

Colonial rule in the Congo began in the late 19th century. King Leopold II of Belgium attempted to persuade the Belgian government to support colonial expansion around the then-largely unexplored Congo Basin. Their ambivalence resulted in Leopold's creating a colony on his own account. Nation set up under Belgian control; quickly became on of the largest scandals at the turn of the Century; the Belgian government would take over the free state and turn it into the Belgian Congo, curbing some of the worse offenses, such as forced labor. Most of wealth taken from the Congo went to public works and development in Belgium. It is important to consider the fact that the Belgian government did not originally desire to claim the Congo as a Belgian colony. The Belgians did not try to resist the Congolese because they saw no reason to. They'd already found reasonable success in what they'd already taken out of the country. There were Belgians who believed that they had done a great favour of "taking on the white man's burden" for the people of the Congo to bring them to civilization. The Belgians had no form of acknowledgement for any form of oppression or brutal abuse of their powers upon the people of the Congo. Despite the fact they promised to help the Congo adjust to its newly formed independent state, they did little to prepare the Congo as a country under European infrastructures. Congo Free State/Belgian Congo Life: - New borders cut through a number of existing African states. - Laws prevented Africans from travelling freely across borders and practicing non-European religions. - Many worked extracting the Congo's riches (ivory, cobalt, rubber, etc.), others worked in harsh conditions in the mines, and others were recruited for the Force Publique, a force that maintained order by inditidating and abusing local populations. - Racially segregated and highly unequal (lack of proper healthcare, education, jobs, etc.).

Compare and contrast China's experience with colonialism and Japan's experience with colonialism

Japan and China reacted to European contact in ways that were mostly rather different. The two countries both thought the Europeans were culturally inferior to themselves, but the Japanese were much more willing to learn about European science and technology. Both the Chinese and the Japanese felt that the Europeans were barbarians. They were particularly repelled by the smell of these foreigners who ate much fattier diets and who did not typically wash very often. Embracing modern technologies and letting go of traditional ways. China lost its independence because it held on. Japan had a successful industrialization that only took a decade. Russia was less successful than Japan, The Ottoman empire had no big attempts at industrialization, and the chinese sought to blend chinese cultural traditions with European Industrial technology. Japan had a successful industrialization that only took a decade. Russia was less successful than Japan, The Ottoman empire had no big attempts at industrialization, and the chinese sought to blend chinese cultural traditions with European Industrial technology. Both China and the Ottoman Empire became more reliant on Western finance than Japan. Both China and the Ottoman Empire experienced occupation of some of their territory by Western military forces; Japan did not. China, the Ottoman Empire, and Japan all were forced by Western powers to sign unequal treaties or capitulations, but Japan eventually was able to renegotiate its treaties in its favor. All three launched modernization programs, but Japan's was more thorough and more successful than those of China and the Ottoman Empire, turning Japan into a modern, united, industrial nation. A number of factors can explain the differences in how they experienced Western imperialism, including the amount of internal strife within each state, the strategic and economic importance to European powers of the Ottoman Empire and China as compared to Japan, and the relatively late and fortuitous timing of Japan's interactions with Western powers. Despite contact with Europe since the 1600s, both China and Japan had actively resisted adopting and European ideas or, with a few exceptions, technologies. However, after Japan was forcibly opened to the west in the 1840s, the Japanese government engaged in a massive and well organized program of modernization. China, on the other hand, continued to view western ideas with suspicion and contempt and tried to maintain their traditional way of life amidst a changing world. Even though both countries were reluctant to modernize at first, Japan's modernization caused it to go from a third-world country to a major world player; whereas China, who refused to modernize, went from being one of the greatest empires in the world to being a resource to be divided amongst the more powerful nations. Both China and Japan had long considered the Europeans to be barbarians and were not interested in goods or ideas from Europe. When the Portuguese first showed up off the coast of China trying to establish trade relations, the Chinese ran away from them calling the Europeans "Ocean Devils." The Japanese similarly referred to the Europeans as "gaijin," which roughly translates as "hairy (smelly) barbarians." When George III of England sent ambassadors and sample trade goods hoping to open trade with China, the Emperor thanked him for his "tribute" but reminded him that, "I rule the most perfect kingdom on earth and have no need of your trinkets." While the quarrelsome Japanese nobles might have been interested in trading with the Europeans for firearms, they were very suspicious of the Jesuits who followed the traders, trying to convert the Japanese to their foreign religion. In China, the final break came when the Jesuit missionaries quarreled with the Emperor over the practice of ancestor worship, causing the Emperor to expel the Jesuits along with most other Europeans. Fearing the influence of the Jesuits, the Japanese enacted the Policy of Seclusion which closed Japan to all foreign contact for almost 250 years. Unfortunately, as Western power grew so did their desire to use Japan as a resource and market. After Japan was forcibly reopened to the west in the 1840s, the Japanese government decided that in order to protect their country they would have to modernize. This meant the modernization of their government, economy, industry, education and especially military. In an attempt to gain knowledge about the west, but still limit foreign influence in Japan, the Japanese government picked their best and brightest to attend school in Europe and the United States. These specialists were to learn everything they could about their subject and then return to Japan to pass on their knowledge. The Japanese used this information to replace their old feudal constitution with a modern parliamentary one. In order to make the most of Japan's limited resources, the government created zaibatsus, massive corporations that were granted complete control of certain portions of the Japanese economy. The Japanese created a modern university system, for both research and learning, modeling it after the system in the United States. The Japanese army was reorganized along the lines of the German army, the most powerful, best equipped and efficient in Europe at the time. Being an island nation, the Japanese naturally modeled their navy after Britain's Royal Navy, also the most powerful in the world. They even went so far as to have their naval academy duplicate the British Royal Naval Academy and built it out of imported English red brick. Japan used this newfound military and economic might to defeat first the Chinese in 1894 and then the Russians in 1905 putting the world on notice that they were a rising power in the Pacific. Despite their successes, Japan's mainland neighbor continued to refuse to modernize. Despite the advances made in Europe and Japan, the Chinese continued to believe in their inherent superiority and saw no need or reason to adopt modern ways, with disastrous consequences for their country. China's government, civil service and military were organized around the ancient philosophies of Confucius, not on modern principals of science or organization. Beginning in the 1850s, the Chinese were forced to accept a series of humiliating treaties which were forced on them by the Europeans. Their loss in the first Opium War forced the Chinese to accept a treaty that ceded Hong Kong to England, permanently opened six ports to English trade and granted all English subjects extraterritoriality in all the treaty ports. Their defeat in the Second Opium War opened eleven more ports to the Europeans, allowed European merchants access to the Yangtze River, opened China to Christian missionaries and allowed the Europeans to establish a permanent presence in Beijing. After China suffered a humiliating defeat at the hands of the Japanese in 1894, the young Chinese Emperor attempted to begin a modest program of reform and modernization. However, this threatened the traditional power structure of the Chinese court and the Emperor was deposed in a palace coup lead by his aunt, the Empress Dowager Xi Ci. Because of the chaos caused by the coup, the Chinese government was not able to maintain order during the peasant uprising known as the Boxer Rebellion. The rebellion gave the European powers the chance to intervene with their militaries, especially when the Boxers attacked the European embassies in Beijing. In 1901, the Chinese were forced to sign the Boxer Protocols which allowed the Europeans to station troops in Beijing and along the railway line between Beijing and the ocean. It also forced the Chinese to pay an indemnity that was equal to one ounce of silver for every man, woman and child in China. It was obvious that China was now simply viewed as a carcass to be carved up by the Europeans and Japanese. These differences in modernization policies had important but divergent effects on China and Japan. The Japanese overcame their resistance to outside ideas and set out on a well organized course of modernization and within fifty years were seen as a major power. China, once seen as the greatest empire in the world, refused to modernize and fell victim to its more advanced competitors. In 1921, the major world powers met in Washington to discuss naval arms limitations. At this meeting, Japan was tacitly acknowledged as the third most powerful naval power in the world. The Chinese people, convinced that their government had failed them, staged a revolution in 1912 that overthrew the Q'ing dynasty and turned China into a republic. Unfortunately, the Japanese continued to follow the European model. When Japan ran out of natural resources, they did what the Europeans always had: conquer another country and take their resources. This brought Japan into conflict with other colonial powers, raising the tensions in the Pacific that eventually led to Japan's involvement in World War II. In China, the Republic soon degenerated into a multi-sided civil war that pit the ruling government against the Chinese Communist Party and various ambitious warlords. The Communists had the ultimate victory in 1948, turning China into a dictatorship but finally allowing it to begin to modernize.

Commodore Perry

Matthew Calbraith Perry (April 10, 1794 - March 4, 1858) was a Commodore of the United States Navy and commanded a number of ships. He served in several wars, most notably in the War of 1812 and the Mexican-American War (1846-48).

Berlin Conference

The Berlin Conference of 1884-85 was a meeting between European nations to create rules on how to peacefully divide Africa among them for colonization. The conference was convened by Portugal but led by Otto von Bismarck, chancellor of the newly united Germany. The Berlin Conference 1884-1885 was one of the defining moments of African history for a number of reasons, the most important being that it changed (for the worse) political boundaries of Africa. Before the Berlin Conference, European countries were already setting up colonies abroad.The Berlin Conference of 1884-85, also known as the Congo Conference (German: Kongokonferenz) or West Africa Conference (Westafrika-Konferenz), regulated European colonization and trade in Africa during the New Imperialism period, and coincided with Germany's sudden emergence as an imperial power. Berlin Conference (1884-85) was held by the European nations to scramble Africa among themselves with the aid of diplomacy or by weapons. The conference had positive as well as negative effects. Africans had lost their lands. Almost 90% of the African continent came under the control of Europeans.

How did the Boxer Rebellion change the way Europeans and Chinese related to each other?

The Boxer Uprising (1899-1901) was a key event in the history of China, as it reflected the colonial powers increasing influence in Chinese affairs and the weakness of late 19th century China. The rebellion consisted of an uprising led by a group known as the Boxers, which culminated in a siege of representatives of the eight colonial powers in Beijing. The rebellion began in the Shandong province of China before leading to Beijing and spreading throughout much of the northern countryside, with many foreigners and converted Chinese Christians attacked. The Boxers were mostly peasants who performed a type of shaman act that has come down through Western interpretation as a sort of boxing, giving rise to the term Boxers. While it is often believed the Boxers were not supported by or were against the main Chinese court and government at the time, the fact is once the uprising was underway it was indeed supported by the Qing Dynasty and its army. The origins of the conflict can be traced to increased foreign presence in China, particularly by Great Britain, the United States, Russia, Germany, France, Italy, Austria-Hungary, and eventually Japan. Mass conversion of locals to Christianity and seizure of property and areas led to increase resentment, while the local population was very poor and the region densely populated. A drought and failed harvest in the great northern plain in Shandong then proved to be the final spark after a period of increased violence against foreigners and Chinese Christians that then that led to the outward expression of anger and mass mobilization. Just 14 years before the Great War, this event also proved to be one of the last instances that all of the great powers of the day were united.

Japanese industrialization

The Meiji Restoration (Meiji Ishin), also known as the Meiji Ishin, Renovation, Revolution, Reform, or Renewal, was an event that restored practical imperial rule to Japan in 1868 under Emperor Meiji. Like European countries, many in the Japanese government turned expansion into a systematic goal, for security, national pride, resources for industrialization, settlement of overpopulation, and markets for manufactured goods. These goals were often intertwined.

What were the consequences of Japan's war with Russia?

The Russo-Japanese War developed out of the rivalry between Russia and Japan for dominance in Korea and Manchuria. In 1898 Russia had pressured China into granting it a lease for the strategically important port of Port Arthur (now Lüshun), at the tip of the Liaodong Peninsula, in southern Manchuria. Russia thereby entered into occupation of the peninsula, even though, in concert with other European powers, it had forced Japan to relinquish just such a right after the latter's decisive victory over China in the Sino-Japanese War of 1894-95. Moreover, in 1896 Russia had concluded an alliance with China against Japan and, in the process, had won rights to extend the Trans-Siberian Railroad across Chinese-held Manchuria to the Russian seaport of Vladivostok, thus gaining control of an important strip of Manchurian territory. However, though Russia had built the Trans-Siberian Railroad (1891-1904), it still lacked the transportation facilities necessary to reinforce its limited armed forces in Manchuria with sufficient men and supplies. Japan, by contrast, had steadily expanded its army since its war with China in 1894 and by 1904 had gained a marked superiority over Russia in the number of ground troops in the Far East. After Russia reneged in 1903 on an agreement to withdraw its troops from Manchuria, Japan decided it was time to attack. The war began on February 8, 1904, when the main Japanese fleet launched a surprise attack and siege on the Russian naval squadron at Port Arthur. In March the Japanese landed an army in Korea that quickly overran that country. In May another Japanese army landed on the Liaodong Peninsula, and on May 26 it cut off the Port Arthur garrison from the main body of Russian forces in Manchuria. The Japanese then pushed northward, and the Russian army fell back to Mukden (now Shenyang) after losing battles at Fu-hsien (now Wafangdian) (June 14) and Liaoyang (August 25), south of Mukden. In October the Russians went back on the offensive with the help of reinforcements received via the Trans-Siberian Railroad, but their attacks proved indecisive owing to poor military leadership. SIMILAR TOPICS Sino-Japanese War World War II Syrian Civil War World War I American Civil War Vietnam War Korean War American Revolution Crimean War Persian Gulf War The Japanese had also settled down to a long siege of Port Arthur after several very costly general assaults on it had failed. The garrison's military leadership proved divided, however, and on January 2, 1905, in a gross act of incompetence and corruption, Port Arthur's Russian commander surrendered the port to the Japanese without consulting his officers and with three months' provisions and adequate supplies of ammunition still in the fortress. The final battle of the land war was fought at Mukden in late February and early March 1905, between Russian forces totaling 330,000 men and Japanese totaling 270,000. After long and stubborn fighting and heavy casualties on both sides, the Russian commander, General A.N. Kuropatkin, broke off the fighting and withdrew his forces northward from Mukden, which fell into the hands of the Japanese. Losses in this battle were exceptionally heavy, with approximately 89,000 Russian and 71,000 Japanese casualties. The naval Battle of Tsushima finally gave the Japanese the upper hand in the conflict. The Japanese had been unable to secure the complete command of the sea on which their land campaign depended, and the Russian squadrons at Port Arthur and Vladivostok had remained moderately active. But on May 27-29, 1905, in a battle in the Tsushima Strait, Admiral Tōgō Heihachirō's main Japanese fleet destroyed the Russian Baltic Fleet, which, commanded by Admiral Z.P. Rozhestvensky, had sailed in October 1904 all the way from the Baltic port of Liepāja to relieve the forces at Port Arthur and at the time of the battle was trying to reach Vladivostok. Japan was by this time financially exhausted, but its decisive naval victory at Tsushima, together with increasing internal political unrest throughout Russia, where the war had never been popular, brought the Russian government to the peace table.

"Unequal treaties"

Unequal treaty is the name given by the Chinese to a series of treaties signed with Western powers during the 19th and early 20th centuries by Qing dynasty China and late Tokugawa Japan after suffering military defeat by the foreign powers or when there was a threat of military action by those powers. Treaties imposed on the qing dynasty by the western forces that had defeated them.

What changes in cities and in your day to day life took place as a result of urbanization?

Urbanization is the process of a population shift from rural areas to cities, often motivated by economic factors. During the industrial era, cities grew rapidly and became centers of population and production. The growth of modern industry from the late 18th century onward led to massive urbanization and the rise of new, great cities, first in Europe, and then in other regions, as new opportunities brought huge numbers of migrants from rural communities into urban areas. In 1800, only 3% of the world's population lived in cities. Since the industrial era, that figure, as of the beginning of the 21st century, has risen to nearly 50%. The physical growth of urban areas as a result of rural migration and even suburban concentration into cities. Urbanization is the process of a population shift from rural areas to cities. During the last century, global populations have urbanized rapidly. Urbanization may be driven by local and global economic and social changes, and is generally a product of modernization and industrialization. Urbanization has economic and environmental effects. Economically, urbanization drives up prices, especially real estate, which can force original residents to move to less-desirable neighborhoods. Urbanization tends to correlate positively with industrialization. With the promise of greater employment opportunities that come from industrialization, people from rural areas will go to cities in pursuit of greater economic rewards.

How did Ethiopia successfully resist European expansion when the Congo did not?

Ethiopians were the only African nations that successfully resisted the Europeans. Ethiopian forces successfully defeated the Italians and kept their nation independent. Afterwards menelik continue to stockpile rifles and other weapons in case of another foreign power that challenge Ethiopian liberty. Only Ethiopia and Liberia remained independent. The one nation in Africa that successfully resisted European rule was Ethiopia. Ethiopia was successful and resisting European take over; the Christian kingdom of Ethiopia was modernized to the extent of being able to defeat European armies Ethiopia was the only nation side from Liberia to preserve its independence. - Built up a large arsenal of weapons that were supplied to them by the Russians as well as by the French, because the Russians and the French, they did not want the Italians arriving. - The Ethiopians who were successfully able to defeat the Italians at the Battle of Adwa. - Ethiopia was able to remain independent because they had modern weapons supplied by the French and the Russians. - At the Battle of Adwa, the Ethiopian army outnumbered the Italians by 7 to 1. And the European rulers, they also signed treaties that recognized Ethiopia as a modern nation. Leopold II was king of Belgium. He ruled with "absolute power" and was very corrupt and extremely dangerous to the people he ruled over. Leopold and other wealthy Belgians exploited the riches of Congo, including its copper rubber and Ivory; they abused workers and locals and did everything bad you can think of - Eventually Europe got mad and made him hand over his colony to the government; things got better-but not that much better. - He took personal control of the Congo region of Africa after the Berlin Conference, and he made it his personal colony - King Leopold, he enslaved the population in order to gather the rubber and ivory that was created to supplement his personal fortune. - Rubber was extremely desirable, so he set quotas, meaning this village must bring in 100-- let's say 100 pounds-- of rubber per day. - For each pound or for each item of ivory that the village was unable to bring in, one villager would either lose a hand or lose a foot for each item that was missed. Historians believe that between 10 million and 20 million people living in this area may have died during the King Leopold's reign. - In 1908, the Belgian parliament had to annex or take this territory away from King Leopold. - They renamed it the Belgian Congo, but they did not have any investigations into these atrocities. - King Leopold II of Belgium hired Henry Stanley to explore the Congo River basin and arrange trade treaties with the African leaders - These efforts were supposed to be for helping people; instead they were more for conquest and profit - Then everyone else wanted colonies (Britain France Germany...)

Steam Engine

James Watt: This person invented the improved steam engine, which enabled rapid development of efficient semi-automated factories on a previously unimaginable scale in places where waterpower was not available. The introduction of this greatly facilitated the removal of water from mine shafts and enabled the shafts to be made deeper, enabling more coal to be extracted. Machine tools: The development of these items, such as the lathe, and planing and shaping machines, enabled all the metal parts of the steam engine to be easily and accurately cut and, in turn, made it possible to build larger and more powerful engines. The invention of this item was the most important innovation for the Industrial Revolution. The steam engine was the most important technological innovation because the steam engine was improved so that it was able to drive machinery and could spin and weave cotton. Because steam engine was powered by coal factories it didn't have to be next to a river anymore but could be anywhere. This also caused cotton production to increase drastically.

Reasons for European Imperialism

New Imperialism. With the wealth of the Industrial Revolution burning in their pockets, the powerful nations of Europe were ready to formally expand their empires into Asia and Africa. ... In this period of New Imperialism, Europeans began to seek formal political control over foreign and overseas areas. Imperialism can give native peoples from different countries many positive opportunities and the change to live a new improved lifestyle. Imperialism can change laws giving people more freedom and rights. ... Although imperialism led to other good results, it also caused many negative situations and events such as slavery. In historical contexts, New Imperialism characterizes a period of colonial expansion by European powers, the United States, and Japan during the late 19th and early 20th centuries. European nations wanted to control lands that had raw materials that they needed for their industrial economies and to open up new markets for their goods that they made, this led to European Imperialism. Imperialism was a desire to create overseas empires.

Why are African countries so often fraught with ethnic violence now?

Nigeria and South Africa could be likened to the Biblical Aaron and Moses, who were endowed with the responsibility to bring Africa out from the bondage of despair, decline and underdevelopment. As regional powers, history has imposed on them the enormous task of finding solutions to some of the most pressing African concerns. African countries today face greater challenges to peace and stability than ever before. The countries of sub-Saharan Africa, including Sierra-Leone, Ivory Coast, Liberia and the Democratic Republic of Congo, are a volatile mix of insecurity, instability, corrupt political institutions and poverty. Alarmingly, most of these countries lack the political will to maintain previous peace agreements, and thus have fallen prey to continuous armed ethnic conflict. (Monty Marshall, 2003) This is partly due to ineffective conflict management. The conflicts in these countries are mostly between ethnic groups, not between states. If not checked, ethnic conflicts are contagious and can spread quickly across borders like cancer cells. Ted Gurr and Monty Marshall have written that most African conflicts are caused by the combination of poverty and weak states and institutions.

Congo Free State

The name given to the territory in central Africa controlled exclusively by Leopold II. It was formally created in 1885 by the Berlin Conference and would remain this way until 1908 when the Belgian government took possession.

British East India Company

When was this company happening? 1600-1873 What are the official names of this company? Governor and Company of Merchants of London Trading into the East Indies(1600-1708), or United Company of Merchants of England Trading to the East Indies (1708-1873) Why was it formed? To get the trade in India and Indonesia What happened after the Spanish Armada was destroyed? After the Spanish Armada was destroyed by Sir Francis Drake in 1588 in the English Channel, English Ships had more latitude to explore and maneuver in the Atlantic and eventually sailed over to the Indian and Pacific Oceans. What happened when it reached India? It won wars against the Portuguese trading companies, and thus won a spot to trade permanently in India and was officially recognized by the Mughal Empire. How did they get power over India? Eventually it would use its power and influence in India to subvert and usurp the Mughal Emperors by conquering lands and forging alliances with the fragmented and weak principalities of the loosely configured Empire, very similar in the nature to the Holy Roman Empire of Charles V. What war allowed for Britain to take off with their trade and be dominant? French and Indian War/ Seven Years War What was the trading relationship with the Mughal Empire? Mughals would have granted the company space and then the companion would send trading goods back to Britain but the more they needed more employees and that got expensive so they just started hiring a whole bunch of Indians. What were the hired Indians called? Sepoy Soldiers

"Civilizing Mission" or "White Man's Burden"

"Civilizing Mission" or "White Man's Burden" is a rationale for intervention or colonization, purporting to contribute to the spread of civilization, and used mostly in relation to the Westernization of indigenous peoples in the 19th and 20th centuries. It was notably the underlying principle of French[citation needed] and Portuguese[citation needed] colonial rule in the late 19th and early 20th centuries. It was influential in the French colonies of Algeria, French West Africa, and Indochina, and in the Portuguese colonies of Angola, Guinea, Mozambique, and Timor. The European colonial powers felt it was their duty to bring Western civilization to what they perceived as backward peoples. Rather than merely govern colonial peoples, the Europeans would attempt to Westernize them in accordance with a colonial ideology known as "assimilation".

Treaty of Portsmouth

(1905) ended the Russo-Japanese War (1904-1905). It was signed in Portsmouth, New Hampshire, after negotiations brokered by Theodore Roosevelt (for which he won the Nobel Peace Prize). Japan had dominated the war and received an indemnity, the Liaodong Peninsula in Manchuria, and half of Sakhalin Island, but the treaty was widely condemned in Japan because the public had expected more. officially ended the Russo-Japanese war. Forced Japan to drop demands for a cash indemnity and russian evacuation of skhalin island, though it received control of Korea.

King Leopold

King of Belgium who ruled an area later to be known as Congo Free State as "King-Sovereign" & hired Stanley to explore the Congo River Basin. Died in December of 1909. Conference that German chancellor Otto von Bismarck called to set rules for the partition of Africa. It led to the creation of the Congo Free State under King Leopold II of Belgium. King of Belgium. He was active in encouraging the exploration of Central Africa and became the ruler of the Congo Free State (to 1908). (p. 732)

Karl Marx

A German philosopher who believed that the bourgeoisie had all the power and the proletariat had none. He believed that the only way to handle the situation was for the Proletarians to rise up and become violent. He thought that in doing so, they would acquire the power that the bourgeoisie had and the government would eventually fade away. Marxism: The political, economic, and social theories of Karl Marx, which included the idea that history is the story of class struggle and that ultimately the proletariat will overthrow the bourgeoisie and establish a dictatorship en route to a classless society. A German journalist who created a radical type of socialism called Marxism and also created communism, which he called the purest phase of socialism. He and Frederich Engels, a German whose father owned a textile mill in Manchester, co-wrote "The Communist Manifesto". He condemned the ideas of Utopians in the 1840s. Also distrusted religion and saw it as the "Opium of Masses". Marx believed that capitalism would destroy itself and eventually a classless society would develop and government or the state would fall. Marx created a new theory called "scientific socialism" that was based on a scientific study of history. These ideas evolved into the common practice known today as communism.

How did the unification of Germany increase the desire for people to create colonial empires?

A MAJOR factor in bringing about the unification of Germany was - Prussian military power A direct consequence of the unification of Germany was that - Italy completed its unification by annexing Rome The ambitions of which leader were exploited to bring about the unification of both Italy and Germany? emperor of France Germany and Italy became unified under a policy of Realpolitik, a German expression that means that the goals of government policy should be accomplished by - the calculated use of power German Empire 1871 After Franco-Prussian War and their victory over France Germany was the dominant power in Europe - Industrial giant of European continent Forces against German unification - Austria was a german state and had more in common with the south german states, may make them want to unite under austria instead opposition of smaller german states religion, half catholic half protestant France wanted Alsace Lorraine and didn't want Germany to be to powerful Forces for German unification - Common language Common customs and traditions Shared history Napoleons influence Economic gain Balance in power - Germany became too powerful

Why did Britain industrialize, but not China?

A final factor was the enclosure movement. British landlords had been able to push their peasants off the land. Those peasants then became a large supply of labor that could be used to work in the new factories that were springing up. All of these factors helped make Britain the first country to industrialize. Great Britain became a powerful empire because it was the birthplace and leading force in the Industrial Revolution, which was a cultural and economic shift from home-based production, traditional agriculture, and manual labor to a system of factory-based manufacturing that included complex machinery, continual technological growth, new energy sources, and developments in transportation. As the Industrial Revolution took hold, Great Britain turned its attention from the rural home to the urban factory and from human power to mechanical power, and it grew so wealthy that it was able to expand and extend its influences across seas and continents. Britain in the mid-18th century and created just the right environment for the rise of industrialization. China has a glorious past in its scientific achievements. And yet they were never able to turn it into economic growth as the West did. If you look at Europe and China in the 19th century, Europe is advancing at breathtaking speed. It's building a rail network, steamships, factories. By the early 20th century, China looked like it was going to be completely occupied by imperialist powers. Clearly the technological and economic development of East and West diverged from 1850 on. China and Europe are different in many ways, but one is that after the Mongol conquest in the 12th century, China remains a unified empire run by a single Mandarin bureaucracy. There is nothing that competes with or threatens China. China does get invaded by Manchu tribes in 1644, but they don't change the structure of the state. They learned to speak Chinese, dress like Chinese and eat like Chinese.

What is Imperialism?

A policy in which a strong nation seeks to dominate other countries poitically, socially, and economically. What were the economic causes of imperialism? - Raw materials for industrial revolution, trade, cheap labor, new markets What new technologies allowed imperialism to take place? - 1st machine gun (Maxim gun), new medicine (1st malaria pill), steam engine (boats could get into the interior of Africa) What were the positive consequences of imperialism? - Economic growth, improved education and healthcare, What were the negative consequences of imperialism? - Native people lost control of their land, destroyed families and culture, slavery, increased local warfare, current political unrest What were the political causes of imperialism? - Nationalism (pride and prestige), national security/border protection, military bases & source of troops What were the social causes of imperialism? - Missionary (spread Christianity), social Darwinism; spread cultural values - racism/cultural superiority; white man's burden What forces enabled imperialism? - Maxim gun, railroad/steamships, cure for malaria, internal divisions, industrial revolution

Sepoy Rebellion

The revolt of Indian soldiers in 1857 against certain practices that violated religious customs; also known as the Sepoy Mutiny. (p. 661) (thank you user for this definition). Pan-Indian nationalism was an attempt to bring India together after the failure of the Sepoy Rebellion. The cause of the Sepoy Rebellion was the Britain had land on almost every continent, and on the guns there is a greased cartridge with pig and cow fat which goes against the Hindu and Muslim religious beliefs - the Sepoys to refuse to bite it off which leads to their arrests and this threw the EIC into crisis. The Sepoy Rebellion is known as the Indian Rebellion of 1857.

Eugenics

The science of improving a human population by controlled breeding to increase the occurrence of desirable heritable characteristics. Developed largely by Francis Galton as a method of improving the human race, it fell into disfavor only after the perversion of its doctrines by the Nazis.

Sino-Japanese War

CAUSE: 1894, - China broke the hands off agreement they made with Japan (in 1885) that they would not send armies to Korea ACTIONS: - Rebellions started happening and Korea's king asked for China's help - The Chinese went to Korea, but the Japanese protested and sent troops to Korea as well - War lasted a few months EFFECTS: - Japan drove China out of Korea - Japan destroyed the Chinese navy - Japan gained foothold in Manchuria - 1895, China and Japan signed a peace treaty ............the treaty gave Japan its first colonies (Taiwan and the Pescadores Islands) China vs. Japan. Japan wins and gains Taiwan as first colony. Japan's victory over China changed the world's balance of power. Both Japan and China wanted Korean territory→ Japan and China pledged to not send their armies to Korea but China broke the agreement→ Japanese troops went to Korea to fight Chinese (Sino-Japanese War) → Japan drove China out of Korea, destroyed its navy and gained a foothold in Manchuria.

Describe the process by which Japan industrialized and "embraced" capitalism?

At first, production was slow, but then Japan opened its ports and began to participate in trade again. Japan had a centralized government, the army was improved, and there was population growth. Pre-World War I Japan was far from the West's equal. It depended on imports of Western equipment and raw materials such as coal; for industrial purposes, Japan was a resource-poor nation. Although economic growth and careful government policy allowed Japan to avoid Western domination, Japan was newly dependent on world economic conditions and was often at a disadvantage. It needed exports to pay for machine and resource imports, and these in turn took hordes of low-paid workers. Silk-production grew rapidly, the bulk of it destined for Western markets. Much of this production was based on the labor of poorly-paid women who worked at home or in sweatshops, not in mechanized factories. Some of these women were sold into service by farm families. Efforts at labor organization or other means of protest were met by vigorous repression. The signs of social stress consisted of: Poor living standards and crowed cities; Many Japanese conservatives resented the passion other Japanese displayed for Western fashions; Disputes between the clashing generations occurred-(Old clung to their conservative ways and traditional standards while the young were more interested in Western styles); Political parties in Japan's parliament clashed with the Emperor's ministers over rights to determine policy; Assassinations and attempted assassinations reflected grievances, including direct action impulses in the samurai tradition; Tension in the intellectual field occurred as well-(Some scholars copied Western philosophers while other expressed pessimistic views about the loss of identity in a changing world); Japanese nationalism built on traditions of superiority, cohesion, and deference to rulers, as well as on the new tensions generated by rapid change.

Bismarck

Bismarck was the first of two Bismarck-class battleships built for Nazi Germany's Kriegsmarine. Named after Chancellor Otto von Bismarck, the ship was laid down at the Blohm & Voss shipyard in Hamburg in July 1936 and launched in February 1939. Work was completed in August 1940, when she was commissioned into the German fleet. Bismarck and her sister ship Tirpitz were the largest battleships ever built by Germany, and two of the largest built by any European power.In the course of the warship's eight-month career under its sole commanding officer, Captain Ernst Lindemann, Bismarck conducted only one offensive operation, in May 1941, codenamed Rheinübung. The ship, along with the heavy cruiser Prinz Eugen, was to break into the Atlantic Ocean and raid Allied shipping from North America to Great Britain. The two ships were detected several times off Scandinavia, and British naval units were deployed to block their route. At the Battle of the Denmark Strait, the iconic battlecruiser HMS Hood initially engaged Prinz Eugen, probably by mistake, while HMS Prince of Wales engaged Bismarck. In the ensuing battle Hood was destroyed by the combined fire of Bismarck and Prinz Eugen, who then damaged Prince of Wales and forced her retreat. Bismarck suffered sufficient damage from three hits to force an end to the raiding mission. The destruction of Hood spurred a relentless pursuit by the Royal Navy involving dozens of warships. Two days later, heading for occupied France to effect repairs, Bismarck was attacked by 16 obsolescent Fairey Swordfish biplane torpedo bombers from the aircraft carrier HMS Ark Royal; one scored a hit that rendered the battleship's steering gear inoperable. In her final battle the following morning, the already-crippled Bismarck was severely damaged during a sustained engagement with two British battleships and two heavy cruisers, was scuttled by her crew, and sank with heavy loss of life. Most experts agree that the battle damage would have caused her to sink eventually. The wreck was located in June 1989 by Robert Ballard, and has since been further surveyed by several other expeditions.

What were the results of the Opium War?

Britain, after subduing much of India, had a lot of resources they could extract from the subcontinent. One of these was opium poppy, used to make opium - an essential ingredient for painkillers. However, when they tried to sell opium in China, the Chinese shut the ports and hired an honest man (important because he didn't accept bribes) to keep them out. Unwilling to lose one of their best customers so easily, Britain, with some help from France, absolutely crushed China in the First Opium War. The undisciplined horde of Chinese soldiers, some of whom still fought with swords, was no match for the modern British guns. The effect of this war was that China opened her ports to Europeans to an extent never before seen. Opium wasn't the only commodity that could now be traded in China, either. Europeans set up trading posts all throughout the country, along the coast and rivers, where they could sell all manner of things. Importantly, though, was the fact that China was now open to Europeans, and not just their goods. Missionaries flooded China, seeking converts after centuries of being locked out. One convert to Protestant Christianity would later establish his own kingdom of followers in southern China, in what would come to be known as the Taiping Rebellion - costing millions of lives.

How did the Opium War feed stereotypes about Asian people in Europe/the Americas?

By the 1830s, Britain realized it could make up the trade deficit with China by selling Indian opium into the Chinese market, making opium Britain's most profitable and important crop in world markets. Eventually, opium poured into China faster than tea poured into British hands; soon, Chinese merchants, already addicted themselves and buying for an addicted population, paid British opium traders in pure silver. Concerned with the sharp rise in opium addiction and the associated social costs and rise in criminal acts, the Chinese government, led by the aging Manchu dynasty, took action against the British. In 1839, the Chinese destroyed British opium in the port city of Canton, sparking the Opium Wars of 1839- 1842. Easily dominating the backward Chinese forces, the British expeditionary force blockaded Chinese ports, occupied Shanghai, and took complete control of Canton. The 1842 Treaty of Nanking granted Britain extensive trading and commercial rights in China, marking the first in a series of unequal treaties between China and European imperial powers. By the end of the century, after five wars between China and various European powers, France, Britain, Germany, Japan, and Russia held territorial and commercial advantages in their respective spheres of influence. These spheres of influence comprised territories, ports, shipping lines, rivers, et cetera in which one nation held exclusive rights to profits and investment. In 1899, the United States, freshly anointed as an inernational force by its crushing victory over Spain in the 1898 Spanish-American War, objected to the prevalence of spheres of influence. The US advocated and pushed through a new Open Door Policy, an effectively imperial policy that demanded that all nations be given equal and complete rights to Chinese markets. Two things happened in the eighteenth century that made it difficult for England to balance its trade with the East. First, the British became a nation of tea drinkers and the demand for Chinese tea rose astronomically. It is estimated that the average London worker spent five percent of his or her total household budget on tea. Second, northern Chinese merchants began to ship Chinese cotton from the interior to the south to compete with the Indian cotton that Britain had used to help pay for its tea consumption habits. To prevent a trade imbalance, the British tried to sell more of their own products to China, but there was not much demand for heavy woolen fabrics in a country accustomed to either cotton padding or silk. The only solution was to increase the amount of Indian goods to pay for these Chinese luxuries, and increasingly in the seventeenth and eighteenth centuries the item provided to China was Bengal opium. With greater opium supplies had naturally come an increase in demand and usage throughout the country, in spite of repeated prohibitions by the Chinese government and officials. The British did all they could to increase the trade: They bribed officials, helped the Chinese work out elaborate smuggling schemes to get the opium into China's interior, and distributed free samples of the drug to innocent victims. The cost to China was enormous. The drug weakened a large percentage of the population (some estimate that 10 percent of the population regularly used opium by the late nineteenth century), and silver began to flow out of the country to pay for the opium. Many of the economic problems China faced later were either directly or indirectly traced to the opium trade. The government debated about whether to legalize the drug through a government monopoly like that on salt, hoping to barter Chinese goods in return for opium. But since the Chinese were fully aware of the harms of addiction, in 1838 the emperor decided to send one of his most able officials, Lin Tse-hsu (Lin Zexu, 1785-1850), to Canton (Guangzhou) to do whatever necessary to end the traffic forever. Lin was able to put his first two proposals into effect easily. Addicts were rounded up, forcibly treated, and taken off the habit, and domestic drug dealers were harshly punished. His third objective — to confiscate foreign stores and force foreign merchants to sign pledges of good conduct, agreeing never to trade in opium and to be punished by Chinese law if ever found in violation — eventually brought war. Opinion in England was divided: Some British did indeed feel morally uneasy about the trade, but they were overruled by those who wanted to increase England's China trade and teach the arrogant Chinese a good lesson. Western military weapons, including percussion lock muskets, heavy artillery, and paddlewheel gunboats, were far superior to China's. Britain's troops had recently been toughened in the Napoleonic wars, and Britain could muster garrisons, warships, and provisions from its nearby colonies in Southeast Asia and India. The result was a disaster for the Chinese. By the summer of 1842 British ships were victorious and were even preparing to shell the old capital, Nanking (Nanjing), in central China. The emperor therefore had no choice but to accept the British demands and sign a peace agreement. This agreement, the first of the "unequal treaties," opened China to the West and marked the beginning of Western exploitation of the nation. Other humiliating defeats followed in what one historian has called China's "treaty century" (major aspects of the so-called "unequal treaties" were not formally voided until 1943). In 1843, France and the United States, and Russia in 1858, negotiated treaties similar to England's Nanking (Nanjing) Treaty, including a provision for extraterritoriality, whereby foreign nationals in China were immune from Chinese law. To compel a reluctant China to shift from its traditional tribute based foreign relations to treaty relations, Europeans fought a second war with China from 1858-1860, and the concluding Treaty of Tientsin (Tianjin) and Convention of Peking (Beijing) increased China's semi-colonial status. More ports were open to foreign residence and trade, and foreigners, especially missionaries, were allowed free movement and business anywhere in the country. Conflicts for the rest of the century wrung more humiliating concessions from China: with Russia over claims in China's far west and northeast in 1850 and 1860, with England over access to the upper reaches of the Yangtze River in 1876, with France over northern Vietnam in 1884, with Japan over its claims to Korea and northeast China in 1895, and with many foreign powers after 1897 which demanded "spheres of influence," especially for constructing railroads and mines. In 1900, an international army suppressed the anti-foreign Boxer Rebellion in northern China, destroying much of Beijing in the process. Each of these defeats brought more foreign demands, greater indemnities that China had to repay, more foreign presence along the coast, and more foreign participation in China's political and economic life. Little wonder that many in China were worried by the century's end that China was being sliced up "like a melon."

What political and social issues prevented China from fully industrializing?

China's industrial revolution, which started 35 years ago, is perhaps one of the most important economic and geopolitical phenomena since the original Industrial Revolution 250 years ago. The reason is simple: Less than 10 percent of the world's population is fully industrialized. Among the many conflicting views that have emerged to interpret China's rise, two stand out as the most popular and provocative. The first sees China's hypergrowth as a gigantic government-engineered bubble. It is not sustainable and will collapse because China has no democracy, no human rights, no freedom of speech, no rule of law, no Western-style legal system, no well-functioning markets, no private banking sector, no protection of intellectual properties, no ability to innovate (other than copying and stealing Western technologies and business secrets), nor a host of many other things that the West has possessed for centuries and have proved essential for Western prosperity and technological dominance.1 According to this view, the bubble will burst at the expense of China's people and environment. The best answer we have is that it [Kiangnan and China] could not surmount the classic constraints of pre-industrial growth. By the late eighteenth century it faced steeply rising costs for food, fuel and raw materials. Increasing population and expanding output competed for the produce of a more or less fixed land area. The demand for food throttled the increase in raw cotton production. Raw cotton prices probably doubled in the Yangtze delta between 1750 and 1800. The demand for fuel (in the form of wood) brought deforestation and a degraded environment. The escape route from this trap existed in theory. Kiangman should have drawn its supplies from further away. It should have cut the costs of production by mechanization, enlarging its market and thus its source of supply. It should have turned to coal to meet the need for fuel. In practice there was little chance for change along such lines. It faced competition from many inland centres where food and raw materials were cheaper, and which could also exploit China's well-developed system of waterway transport. The very perfection of China's commercial economy allowed new producers to enter the market with comparative ease at the same technological level. Under these conditions, mechanization — even if technologically practical — might have been stymied at birth. And, though China had coal, it was far from Kiangnan and could not be transported there cheaply. Thus, for China as a whole, both the incentive and the means to take the industrial "high road" were meagre or absent.

Direct rule

Colonial government in which local elites are removed from power and replaced by a new set of officials brought from the mother country. When a colony is ruled by the mother country. Soldiers and officials sent to administer the colonies directly. An example of direct rule is: France in West Africa Goal 1: impose french culture on their colonies Goal 2: turn colonies into provinces

Direct Vs. Indirect Rule

Direct: In a direct rule, a colony would be governed by foreign officials while no self-rule was given to natives. The goal was assimilation of the colony into an empire. Government institutions were based on European governments. The Belgian King Leopold II practiced the most brutal style of direct rule in the Belgian Congo. Almost 10,000,000 Congolese were killed in his entire rule. France also often practiced direct rule of her colonies. Vietnam, Algeria, Morocco, and Tunisia are notable examples. A territory or piece of land governed directly by a foreign power. The governing power may exercise direct rule through soldiers and officials, forcing the natives into submission, or the power may exercise indirect rule, administering the colony through the native aristocracy and westernizing the upper-class. Indirect: In an indirectly ruled colony, local government officials might be used, while the native upper-class was given limited self-rule over the colony. The goal was to Westernize future leaders of a colony in order to perpetuate a power's control. This was accomplished in Raj India under the British East India Tea Company until the Sepoy Rebellion. After the rebellion was put down, the British Government assumed direct rule over the colony. A system of government used by the British and French to control parts of their colonial empires, particularly in Africa and Asia, through pre-existing local power structures. These dependencies were often called "protectorates" or "trucial states".

Communism

Economic system where there is NO competition and everyone makes the same amount of money. Everything is shared equally (even if you work harder than someone else). There is no private property - the government owns everything. There is no competition in this economic system. This is where a government controls everything and every part of a person's things. Everyone is equal, and people are paid based upon their ability and their need for the community. A form of socialism that abolishes private ownership; a political theory favoring collectivism in a classless society. Karl Marx: He created the concept of communism (equal everything). Also cofounder of socialism. German journalist and philosopher. Marxist Socialism. Wrote, "The Communist Manifesto (1848)" & "Das Kapital" (Vols. I-III, 1867-1894). Did not agree with earlier socialist movements such as Utopian & believed that the economy influenced Social Strucutre. Preached the necessity of social revolution to create proletarian dictatorship.

Capitalism! How did it address the IR?

Economic system where there is competition and the goal is to make a profit. Laissez-faire: When government does not interfere with business (this happens ONLY in capitalism). There is private property. This economic system aims to make the best products for the lowest amount of money, which often leads to exploitation of workers. The main goal is to make a profit. Businesses follow their own self-interest when they compete against each other. An economy in which trade of raw goods and products where traded and sold by businesses and companies, not the government. Economic system in which the factors of production are privately owned and money is invested in business ventures to make a profit. Replaced feudal empires (i.e. Holy Roman Empire) and monarchies. Created markets where buyers and sellers of goods and services agreed on prices. Created market types: Competition, Monopoly, and Oligopoly. Long distance trade increased dramatically due to innovation in transportation. Individuals and businesses own property and the mean of production. Progress results when individuals follow their own self interest. Businesses follow their own self interest by competing for the consumer's money. Each business tried to produce goods or services that are better and less expensive then those of competitors. Government should not interfere in the economy because competition create efficiency in business. Addressing the IR: - Led to investing in factories - Led to increased production and higher demands for raw materials - Led to world wide trade - Led to new inventions and new innovations - Led to changes in transportation, agriculture, and communication

E.D. Morel

Employee of the Elder-Dempster shipping company based out of Antwerp. Responsible for basically starting the international human rights movement in the Congo Free State. Created the Congo Reform Association and was a constant thorn in the side of Leopold II. He formed his own newspaper 'The West African Mail' and wrote 'Red Rubber' to publicise the atrocities committed by King Leopold II and his officers in the Congo.

What did the British East India Company do when it ruled India-and how did it create famines?

Famine had been a recurrent feature of life the Indian sub-continental countries of India, Pakistan and Bangladesh, and reached its numerically deadliest peak in the late 18th and early 19th centuries. Historical and legendary evidences names some 90 famines in 2,500 years of history in South Asia alone. There are 14 recorded famines in India between the 11th and 17th centuries. Famines in India resulted in more than 60 million deaths over the course of the 18th, 19th, and early 20th centuries. The last major famine was the Bengal famine of 1943. A famine occurred in the state of Bihar in December 1966 on a much smaller scale and in which "Happily, aid was at hand and there were relatively fewer deaths". The drought of Maharashtra in 1970-1973 is often cited as an example in which successful famine prevention processes were employed. Famines in British India were severe enough to have a substantial impact on the long term population growth of the country in the 19th and early 20th centuries. Indian agriculture is heavily dependent on climate: a favourable southwest summer monsoon is critical in securing water for irrigating crops. Droughts, combined with policy failures, have periodically led to major Indian famines, including the Bengal famine of 1770, the Chalisa famine, the Doji bara famine, the Great Famine of 1876-78, and the 1943 Bengal famine. Some commentators have identified British government inaction as contributing factors to the severity of famines during the time India was under British rule. Famine largely ended by the start of 20th century with the 1943 Bengal famine being an exception related to complications during World War II. The 1883 Indian Famine Codes, transportation improvements and changes following independence have been identified as furthering famine relief. In India, traditionally, agricultural labourers and rural artisans have been the primary victims of famines. In the worst famines, cultivators have also been susceptible.

Legacies of Belgian Congo

Human toll- 10-30 million killed more maimed, raped, enslaved Economic toll- destroyed, indigenous trade network and independent tribal economies turned Congo into an export-based monoculture of rubber and minerals social toll- fanned the flames of internal conflict among indigenous African peoples historic toll- the world will never know how African people would have developed if they could have defined themselves

Middle Class/Bourgeoisie

Image result for middle class/bourgeoisie industrial revolution After the Industrial Revolution (1750-1850), by the mid-19th century the great expansion of the bourgeoisie social class caused its stratification - by business activity and by economic function - into the haute bourgeoisie (bankers and industrialists) and the petite bourgeoisie (tradesmen and white-collar workers). Middle class, Marx considered them the oppressors in the eternal class struggles, approx. half of society. Third Estate (97% of the French population): Group 1 - The Bourgeoisie (educated middle class), Group 2 - The Workers, Group 3 - The Peasants. The Industrial Revolution created this new middle class, or bourgeoisie, whose members came from a variety of backgrounds. Some were merchants who invested their growing profits in factories. Others were inventors or skilled artisans who developed new technologies. Some rose from "rags to riches," a pattern that the age greatly admired. Made up of people who built the factories, bought the machines, and figured out where the markets were. Qualities included initiative, vision, ambition, and greed. Faced wretched working conditions, long hours, no job security, and no minimum wage. New middle class began to transform the social structure of Britain. Landowners and aristocrats had one occupied the top position in British society and, with most of the wealth, wielded political and social power. However, factory owners, merchants, and bankers were starting to become more rich than the aristocrats and landowners. Landowners looked down on people who earned their money in the "vulgar" business world and it wasn't until late 1800s when rich entrepreneurs were considered equal to the lords of the countryside. A larger middle class of neither rich or poor emerged. The upper middle class was government employees, doctors, lawyers, and managers of factories, mines, and shops. The lower middle class included factory overseers and skilled workers like toolmakers, mechanical drafters, and printers.

Working Conditions

Immediate results of the Industrial Revolution: Building factories, abundant natural resources, railroads, growth within cities, bad working conditions. There were poor working conditions during that time and their goal was to improve pay and working conditions. Experienced low wages, long hours, unsafe enviroments, and the constant threat of unemployment.

Why do capitalists believe that competition is good?

Individuals and businesses own property and the mean of production. Progress results when individuals follow their own self interest. Businesses follow their own self interest by competing for the consumer's money. ... Government should not interfere in the economy because competition create efficiency in business. Economic and Social are the two type of competitions in Capitalism. The individual owns the factory in capitalism. Wealth or capital is privately owned and invested in business to create more wealth for that investor - Began during the commercial revolution of the 16th century - Led to investing in factories - Led to increased production and higher demands for raw materials - Led to world wide trade - Led to new inventions and new innovations - Led to changes in transportation, agriculture, and communication - English parliament was firmly under the control of the capitalist classes - Increase in agriculture production and turned the established rules of land ownership on their head - Lands previously held in common by tenant farmers changed into large private farms, worked by a smaller labor force - The revolution moved economic power away from the aristocratic classes and into the hands of the new middle class, the bourgeoisie - Economic system in which the factors of production are privately owned and money is invested in business ventures to make a profit - Created a market where buyers and sellers of goods and services agreed on prices - Long distance trade increased dramatically due to innovation in transportation (Think railroads!) - Role of market competition and entrepreneurial abilities - Impact on standard of living and the growth of the middle class - Dissatisfaction with poor working conditions and the unequal distribution of wealth in society

Why did Karl Marx believe the proletariat will overthrow the bourgeoisie? Why did Marx believe this revolution was inevitable?

Karl Marx (1818-1883) was a revolutionary German economist and philosopher, and the founder of the Communist movement. Marx was writing against a backdrop of great industrial change. Overcrowded, newly industrialised cities were expanding, and much of the working class lived in great poverty. Marx saw history as the story of class struggles, in which the oppressed fight against their oppressors. According to Marx, as history unfolded, the victory of one class would pave the way for the future freedom of the rest of society. Marx viewed the unfolding process of history as follows: - First in ancient and medieval society the landed and wealthy had oppressed the slaves and the poorest plebeians and laborers. - Then, as new technologies were invented and market forces grew stronger, everything changed. The middle classes - gaining wealth and power from trade and manufacture - challenged the power and authority of the old rulers. - But at this stage a new struggle was formed between the bourgeoisie (the property owning class) and the proletariat (the industrial working class). Marx argued that the capitalist bourgeoisie mercilessly exploited the proletariat. He recognized that the work carried out by the proletariat created great wealth for the capitalist. The products created in the factory (the material outcome of the workers' labour) were sold for more than the value of the labour itself i.e. more than the workers' wages. For instance, the factory worker may get paid £2 to produce a yard of cloth. The capitalist then sells the cloth for £5. In this way, the capitalist, who controls the process of production, makes a profit. But the worker does not benefit from this added value, and fails to benefit from the fruits of his/her own labour. Marx believed that capitalism contained the seeds of its own destruction. He described how the wealth of the bourgeoisie depended on the work of the proletariat. Therefore, capitalism requires an underclass. But Marx predicted that the continued exploitation of this underclass would create great resentment. Eventually the proletariat would lead a revolution against the bourgeoisie. The final struggle would lead to the overthrow of capitalism and its supporters. Marx wrote that modern bourgeois society 'is like the sorcerer, who is no longer able to control the powers of the nether world whom he has called up by his spells.'

Labor Unions

Labor Unions affected the Industrial Revolution because the Unions could either support a candidate or dislike them, which seemed to have a large effect on their popularity and success in politics. Employers argued that these were dangerous to business and the country. Three Main Goals of Labor Unions: 1. Higher wages 2. Better working conditions 3. Eight hour work day Laborers were working up to 10 to 12 hour days 6 to 7 days a week and for very little wages. Workers were not given safe working conditions. The corruption of businesses and the government forced workers to organize so their voice could be heard.

Factory

Large buildings that wealthy textile merchants set up. they would hold spinning machines. A system in which workers and machines were together in one place to make goods, and the employees worked a set number of hours for a certain amount of money. Worked in big spaces with many people under a supervisor. Only did a small part of the task and did not work on a product from start to finish. Paid wages by based on hours worked and amount of goods produced. Places where machines manufacture and produce goods. Developed as a result of an increase in cotton production, Large expensive machinery made it necessary to operate in one large building, Factories located by rivers to supply water for power for machines, Labor brought to factory to work - cottage industry ended, Division of labor - owners-supervisors-workers, Developed to replace the domestic system of production, Faster method of production, Workers concentrated in a set location, and Production anticipated demand. Mostly made up of immigrants work long 12hr days 6 days a week,worked in sweatshops small hot dirty and dark,mainly woman worked there mass producing items and they were fined is any rules were broken. it was dangerous accidents were common people lost their hearing.

Menelik II

Menelik took important steps to strengthen and modernize his domain. He made Addis Ababa his capital, constructed a railroad, attempted to end the slave trade, and curbed the feudal nobility. His conquests doubled the size of the country and brought the present S Ethiopia (largely Muslim in population) into the realm. Emperor of etheopia who was successfull in keepig his country as the only indepent african nation from european imperialism. Emperor of Ethiopia who played Italians, British, and French against each other while buying weapons from France and Russia. In the Battle of Adowa, Ethiopian forces successfully defeated the Italians and maintained their independence. Emperor of Ethiopia who successfully defeated the Italians and maintained his nation's independence. He kept Ethiopia independent.

How did the Steam Engine begin to change cities and urban development?

New sources of energy were harnessed such as coal, which was used to develop the steam engine. Coal was also used in the production of iron, which was used to construct machines and steam engines. Iron was better quality and less expensive. Thus, it was used more widely to produce steam engines, bridges, railroads, etc. The steam engine allowed for operating machinery, locomotives, and steamships. First machine to transform fossil fuel into mechanical energy. Replaced animal, wind, and waterpower. The invention of the steam engine sparked many later inventions like the sun-and-planet gear which turned the back and forth action of the piston into rotary motion. This allowed steam engines to power machinery in flour and cotton mills, pottery manufacturers, and other industries. No limit to the amount of coal in the ground and steam generated energy seemed to be an inexhaustible source of power. Many aspects of society and economics are present due to the profound impact these inventions had, which is why they are such big turning points in history. With the birth of these revolutionary inventions, different ways of life came about, especially in terms of jobs. For example, people started working by the hour and were paid by the hour, as opposed to working sunrise to sunset, not really having an idea of how much they were making by the hour or how efficient they were in terms of pay. And, since many products became cheaper, people had extra money to use for leisure activities such as going on vacation. Finally, the steam locomotive, because it brought about mass business, paved the way for inventions that changed everyday lives of many Europeans, such as the telephone and electric lighting. No doubt, the steam locomotive played a huge role in transforming the everyday life of countless Europeans. The railroad rapidly spread and sped transportation. This created new jobs, forcing people to move into cities.

Why did the British want to trade more with China?

Opium trade, in Chinese history, the traffic that developed in the 18th and 19th centuries in which Western countries, mostly Great Britain, exported opium grown in India and sold it to China. The British used the profits from the sale of opium to purchase such Chinese luxury goods as porcelain, silk, and tea, which were in great demand in the West. Britain and other European countries undertook the opium trade because of their chronic trade imbalance with China. There was tremendous demand in Europe for Chinese tea, silks, and porcelain pottery, but there was correspondingly little demand in China for Europe's manufactured goods and other trade items. Consequently, Europeans had to pay for Chinese products with gold or silver. The opium trade, which created a steady demand among Chinese addicts for opium imported by the West, solved this chronic trade imbalance.

Rubber trade

Rubber: sure wealth. The Industrial Revolution in Europe led to demand for uses that natural rubber could satisfy. At that time, it was exclusively found in the Amazon Basin. It was a desirable commodity, valued at a high price, and thought to create wealth and dividends for whoever would dare invest in the trade.

Russo-Japanese war

Russia and Japan were fighting over Korea, Manchuria, etc. Began in 1904, but neither side could gain a clear advantage and win. Both sent reps to Portsmouth, NH where T.Roosevelt mediated Treaty of New Hampshire in 1905. TR won the nobel peace prize for his efforts, the 1st pres. to do so. Interests of Japan and Russia in Korea caused the conflict that led to Russo-Japanese war from 1904 to 1905. Japan defeated Russian troops and crushed its navy. By 1910, Japan had complete control of Korea and parts of Manchuria. A conflict that grew out of the rival imperialist ambitions of Russia and Japan over Manchuria and Korea that Roosevelt helped with peace negotiations. CAUSES: - 1903, Japan offered to recognize Russia's rights in Manchuria, if they stayed out of Korea - Russians did not agree - February 1904, Japan, surprise attack on Russian ships that were anchored on the coast of Manchuria ACTIONS: - Japan drove Russian troops out of Korea - Japan captured most of Russia's Pacific fleet, destroyed Russia's Baltic fleet EFFECTS: - 1905, Japan and Russia began peace treaties/negotiations - President Theodore Roosevelt help draft treaty - Treaty of Portsmouth (signed in Portsmouth, New Hampshire) ..........gave Japan the captured terrorist ..........forced Russia to withdraw from Manchuria and to stay out of Korea

Why did Japan colonize Korea, and go to war with Russia?

Since the end of the First Sino-Japanese War in 1895, Japan feared Russian encroachment on its plans to create a sphere of influence in Korea and Manchuria. ... The Japanese government perceived a Russian threat to its plans for expansion into Asia and chose to go to war. The 1905 Korea-Japan Convention had already made Korea a protectorate of Japan. Under the annexation treaty, the Korean emperor handed sovereign power over his country to the Japanese emperor "completely and forever." Thus Korea became a colony of Japan. The Russo-Japanese War was a war between the Japanese Empire and the Russian Empire. It started in 1904 and ended in 1905. The Japanese won the war, and the Russians lost. The war happened because the Russian Empire and Japanese Empire disagreed over who should get parts of Manchuria and Korea. Soon after, they unleashed torpedoes against Russian ships in a surprise attack that began the Russo-Japanese War. The conflict grew over competition between Russia and Japan for territory in both Korea and Manchuria, in northern China.

How Unification of Germany is made possible by Industrial Revolution

Started in the 1800s because Germany was not politically unified until the 19th century. It started about a century later than Britain. Germany imitated lessons learned from the British experience. The establishment of the German Customs Union was the focal point of the German industrialization. Bismarck created a unification that provided a base platform for industry to grow. He implemented several policies to protect baby businesses. Created laws which forced up the costs of many foreign items. All this strengthened the economy and the industry. Villages quickly merged into cities as new factories sprung over night demanding a constant supply of labor. Essen was the first industrial city in Germany and became the home of base for companies taking advantage of the industrial revolution. Germans sent their children to England to learn industrial management. The most important development was the intensive use of hydrocarbon fuels: coal, oil and natural gas. Before using these fuels, they used wind or water power. Another trigger for the German idustrializatins was the development in the railway technology. The rapid expansion of this technology, Germany started to catch up with Britain between 1850 and 1870. Germany made its own locomotives. Germany built railroads that linked growing manufacturing cities such as Frankfurt. At the beginning the products were labeled cheap knock-offs by the world customers, however the price of these products made it more able to sell the British products. Germany imported British equipments and engineers. Germany invested heavily in research, mostly on chemistry, motors and electricity. By 1900s German chemical industry dominated the world market for synthetic dyes. Them, it began to expand in chemistry (pharmaceuticals, photographic film, agricultural chemicals and electrochemicals). Effects: Rapid growing population, high skill working people, huge home market, vast availability of employment. Otto von Bismarck: Prime Minister of prussia, then chancellor of German Empire Responsible for unity in 1871, Otto von Bismarck became chancellor in 1862 and changed the course of history. When the Prussian monarch faced an impasse with liberal forces in the parliament, the Kaiser appointed Bismarck to defend the autocratic government, by manipulating the issue of German unification, he divided and defeated the liberal challenge. Over the next decade Bismarck enlarged the territory of Prussia in the northern half of Germany through a combination of military victories and diplomatic skill, finally, in 1870 Bismarck maneuvered France into declaring war on an obviously stronger German-led alliance. Military victories and the nationalist sentiments stirred by war led to the incorporation of the southern German states into the North German Confederation. Krupp family: The Krupp family (see pronunciation), a prominent 400-year-old German dynasty from Essen, have become famous for their production of steel, artillery, ammunition, and other armaments. The family business, known as Friedrich Krupp AG, was the largest company in Europe at the beginning of the 20th century. The Siemens Brothers: Ernst Werner von Siemens - German electrical engineer who played an important role in the development of the telegraph industry, his name has also been adapted as the SI unit of electrical conductance, the Siemens, he is the founder of the electrical and telecommunications company Siemens; He wanted to prove that Germany was a great world power. Karl siemens - redirected exhaust fumes to superheat our intake-hotter "burns"removes more impurities from molten iron. Karl Wilhelm Siemens - When Wilhelm became kaiser, he was determined to present himself as a bold German warrior who would expand his nation's power. Germany, he declared, must have its "place in the sun" among the world's greatest nations. Germany industrialized after Britain. These made Germany's industrialization more efficient and quicker. Germany quickly began to grow and now Germany is a fully industrialized country.

How did the IR lead to a rising middle class?

The Industrial Revolution made drastic changes on the lives of individuals. Two classes that benefited from it were the "middle" and "upper" classes. These two classes were composed of people that had wealth and success. Even though most could afford goods anyway, the prices lowered even more, so that those who could not afford them before could now enjoy the comfort and convenience of the new products being made. The middle class was composed of businessmen and other professionals. The larger the Industrial Revolution grew, the more powerful these individuals became. Individuals and groups formed new libraries, schools, and universities because there was a sudden need for education (possibly due to the increase in population). The middle and upper classes had better food and housing, which led to fewer diseases and longer living among these groups. Since these classes were treated so well, their population grew and thus had minimal difficulty living during the Industrial Revolution. In contrast with the middle and upper classes, the "working" class was not well off. In the working class, many were replaced in factories by machines. But on the other hand, many also gained new jobs in factories working with machinery. The average adult worker worked quite often: five to seven days of the week, for more than half the day per shift. Children as young as fifteen worked for minimal wages. Some of the children became deformed or crippled due to their work, which was often. Most workers worked for relatively low wages due to their incapability to produce goods. The women and children were not paid as much as the men were. The housing was not desirable either - for example there was frequent overcrowding. The housing had unsanitary features which led to diseases. Workers who were desperate lived near a factory. What also made life difficult during the Industrial Revolution was that there were limited privileges such as few people voted, nor were they allowed to do anything to improve their working condition that was legal. The amount of carbon dioxide increased two-fold as people moved closer to factories hoping to obtain employment. Resources started diminishing, and the use of pesticides and hazardous chemicals began to increase.

What was the Industrial Revolution?

The Industrial Revolution was the transition to new manufacturing processes in the period from about 1760 to sometime between 1820 and 1840. Prior to the Industrial Revolution, which began in Britain in the late 1700s, manufacturing was often done in people's homes, using hand tools or basic machines. Industrialization marked a shift to powered, special-purpose machinery, factories and mass production. The rapid development of industry that occurred in Britain in the late 18th and 19th centuries, brought about by the introduction of machinery. It was characterized by the use of steam power, the growth of factories, and the mass production of manufactured goods. James Watt from Scotland designs a more efficient steam engine. One of the most important inventions of the Industrial Revolution, steam engines power the first trains, steamboats, and factories. Eli Whitney creates a machine that makes it much easier to separate cotton seeds from cotton fiber. For when it came the revolution brought good things with it but also the bad. Pro: The good the Industrial Revolution brought with it were steam engines, more houses, money, and more clothing Con: The bad that is brought with the Industrial Revolution was the heavy swerage, child labour and heaps of pollution. The Industrial Revolution impacted the environment. The world saw a major increase in population, which, along with an increase in living standards, led to the depletion of natural resources. The use of chemicals and fuel in factories resulted in increased air and water pollution and an increased use of fossil fuels. Led to the Industrial Revolution: 1. Industrial Production 2. Agricultural Production 3. Iron Smelting 4. Coal Extraction 5. The Railways 6. Steam Power

McCartney mission

The McCartney Embassy, also called the Macartney Mission, was the first British diplomatic mission to China, which took place in 1793. It is named for its leader, George Macartney, Great Britain's first envoy to China. The unsuccessful attempt by the British Empire to establish diplomatic relations with the Qing Empire.

Taiping Rebellion

The Taiping Rebellion was a widespread civil war in China from 1850 to 1864. The rebellion was run by Hong Xiuquan against the Qing Dynasty. Hong Xiuquan, who was a converted Christian, established the Taiping Heavenly Kingdom. The Kingdom controlled vast parts of southern China and at the peak, they controlled about thirty million people. The goal of the Heavenly Kingdom was to abolish old Chinese costumes such as Buddhism and Confucianism and replace them with Christianity. The Taiping Rebellion is an example of "total war" in modern China. Almost every citizen under their rule was trained for war. In the primary document it says "Each man throughout the empire who has a wife, sons, and daughters amounting to three or four mouths, or five, six, seven, eight, or nine mouths, must give up one to be a soldier." Eventually the Qing Dynasty defeated the rebellion with the help of British and French forces. The rebellion started due to the fact that people saw the Qing rule as ineffective. People in the south, which would include most of the labor force, mainly felt this way. Also there was much interaction between the Chinese and the Western world. The Chinese had to endure defeats like the Opium Wars. Forces started to gather under the rule of Hong Xiuquan whom considered himself a brother of Jesus. The Heavenly Kingdom rulers like Hong Xiuquan were like western crusaders in they way that they wanted to spread Christianity and the word of God.

Urbanization

The act or fact of urbanizing, or taking on the characteristics of a city: Urbanization has led to more air pollution and increasing childhood asthma. Growth of cities. Key Factors that led to the growth of Urbanization: Large number of immigrants coming to the United States and the growth of the industrial base in large cities especially on the east coast. The growth of cities and the migration of people into them. Urbanization is the movement of people to cities. Since there were more people in the cities there was more air pollution, slums, no sanitation, lack of running water and disease became more common.

What does Adam Smith argue is the correct role of government?

The first theme in The Wealth of Nations is that regulations on commerce are ill-founded and counter-productive. The prevailing view was that gold and silver was wealth, and that countries should boost exports and resist imports in order to maximize this metal wealth. Smith's radical insight was that a nation's wealth is really the stream of goods and services that it creates. Today, we would call it gross national product. And the way to maximise it, he argued, was not to restrict the nation's productive capacity, but to set it free. Another central theme is that this productive capacity rests on the division of labour and the accumulation of capital that it makes possible. Huge efficiencies can be gained by breaking production down into many small tasks, each undertaken by specialist hands. This leaves producers with a surplus that they can exchange with others, or use to invest in new and even more efficient labour-saving machinery. Smith's third theme is that a country's future income depends upon this capital accumulation. The more that is invested in better productive processes, the more wealth will be created in the future. But if people are going to build up their capital, they must be confident that it will be secure from theft. The countries that prosper are those that grow their capital, manage it well, and protect it. A fourth theme is that this system is automatic. Where things are scarce, people are prepared to pay more for them: there is more profit in supplying them, so producers invest more capital to produce them. Where there is a glut, prices and profits are low, producers switch their capital and enterprise elsewhere. Industry thus remains focused on the nation's most important needs, without the need for central direction. But the system is automatic only when there is free trade and competition. When governments grant subsidies or monopolies to favoured producers, or shelter them behind tariff walls, they can charge higher prices. The poor suffer most from this, facing higher costs for the necessities that they rely on. A further theme of The Wealth Of Nations is that competition and free exchange are under threat from the monopolies, tax preferences, controls, and other privileges that producers extract from the government authorities. For all these reasons, Smith believes that government itself must be limited. Its core functions are maintaining defence, keeping order, building infrastructure and promoting education. It should keep the market economy open and free, and not act in ways that distort it.

Italy

The imperialism came in Italy in 800, was created after the Kingdom of Italy joined other European powers in establishing colonie overseas during the "Scramble For Africa." Throughout the 1800's numerous European countries started to colonize Africa. The Italians were motivated to colonize some countries in Africa such as Libya, Eritrea, and Italian Somaliland to keep up politically with the other European countries. Italian colonization of Libya. The history of Libya as an Italian colony began in the 1910s and lasted until February 1947, when Italy officially lost all the colonies of the former Italian Empire.

Qing dynasty

The last dynasty of China: (up to 1912) also called the Manchu Dynasty. (Foreigners ruling China.) Characteristics: highly corrupt, inefficient; experienced wars & rebellions (1839 on). Nationalist Revolution overthrew the dynasty. Second foreign dynasty and last dynasty to rule China. From Manchuria, what is today northeast China. Ruled for more than 260 years and expanded Chinese rule over Taiwan, Tibet, Central Asia and Mongolia.

Proletariat

The lower or working class. They were the lower part of the middle class and worked for the higher class or bourgeoisie who had all the power. Under Marxism, they were supposed to rise up against the bourgeoisie and take their power. Workers or working class people. This term is often used in the context of Marxism or communism Marx believed that the proletariat would revolt against the manufacturers and factories, seize factories and mills from the capitalists, and produce what society needed. Marx believed the workers would share their profits and promote economic equality for all. Believed they would control the government in the "dictatorship of the proletariat" until there was peace and harmony and a government was no longer needed. Dictatorship of the Proletariat: A society where the Proletariat or the working class, have the power. This was the type of society that Karl Marx supported. He believed the bourgeoisie had all the power and thought the proletarians should overthrow them. This type of society would be the result of actually doing what Marx believed. The industrial working class; in Marxism, the class that will ultimately overthrow the bourgeoisie. Theory proposed by Marx where, in a communist society, the workers would control the government. Marx believed that the large proletariat, being cheated by capitalism and the Revolution, would revolt and seize the factories and mills from capitalists. The workers would control the government until people began to understand how to live in a communist society and, after a period of cooperative living and education, the state or government would wither away as a classless society developed.

Constitution of 1889

The new constitution was promulgated by Emperor Meiji on February 11 (the National Foundation Day of Japan in 660 BC), 1889, but came into effect on November 29, 1890. The first National Diet of Japan, a new representative assembly, convened on the day the Meiji Constitution came into force. Meiji Constitution, constitution of Japan from 1889 to 1947. After the Meiji Restoration (1868), Japan's leaders sought to create a constitution that would define Japan as a capable, modern nation deserving of Western respect while preserving their own power. The resultant document, largely the handiwork of the genro (elder statesman) Itō Hirobumi, called for a bicameral parliament (the Diet) with an elected lower house and a prime minister and cabinet appointed by the emperor. The emperor was granted supreme control of the army and navy. A privy council composed of the Meiji genro, created prior to the constitution, advised the emperor and wielded actual power. Voting restrictions, which limited the electorate to about 5 percent of the adult male population, were loosened over the next 25 years, resulting in universal male suffrage. Political parties made the most of their limited power in the 1920s, but in the 1930s the military was able to exert control without violating the constitution. After World War II, a U.S.-approved constitution stating that "sovereign power resides with the people" replaced the Meiji Constitution.

Meiji "restoration"

The political program that followed the destruction of the Tokugawa Shogunate in 1868, in which a collection of young leaders set Japan on the path of centralization, industrialization, and imperialism. A change in political control of Japan whereby the emperor regained his political power. The modernization and industrialization of Japan in the 1800's. Meiji is known as "Enlightened rule." Period of Modernization (late 19th c.) After Japan forced open, she began a policy of modernization in order to compete with the rest of the world. Name 3 changes that occurred during the Meiji Restoration: - Political changes: feudalism ends, developed constitution and central government, modernized military - Social changes: adopted western ways/culture - Adopted universal public education - Maintains traditional cultural values - Economic: built railroads;factories;mined coal - Land reforms: feudal lords give up their land to the emperor

What criticisms did the British have about the Chinese government?

The primary motive of British imperialism in China in the nineteenth century was economic. There was a high demand for Chinese tea, silk and porcelain in the British market. However, Britain did not possess sufficient silver to trade with the Qing Empire. Thus, a system of barter based on Indian opium was created to bridge this problem of payment. The subsequent exponential increase of opium in China between 1790 and 1832 brought about a generation of addicts and social instability. Clashes between the Qing government and British merchants ultimately escalated into the infamous Opium Wars. As a result, the British were given the island of Hong Kong and trading rights in the ports of Canton and Shanghai. Although British imperialism never politically took hold in mainland China, as it did in India or Africa, its cultural and political legacy is still evident today. Honk Kong remains a significant center of global finance and its government still functioned in much of the same ways as it did under British colonialism. Furthermore, the language of English and British culture highly impacted the society of Hong Kong and Southern China for over a century. This Research Guide is divided into four main components. The first section is devoted to the definitions and qualifications of imperialism. This part mainly consists of print sources that focus on the political, economic and social mechanisms of imperialism. It provides scholarly perspectives and criticisms regarding its causes and effects. The second section consists of both print and interactive sources. This section focuses on the topic of British Imperialism in China from a British perspective. The sources include various political justifications and financial factors that influence Britain's diplomatic decisions and imperialist tactics. The third section presents the Chinese perspective. The sources in this section explain the development of Chinese nationalism and the intricacies of international relations in the Qing court. The final section deals with the legacy of British imperialism in Hong Kong and southern China. The sources here examine the cultural and political footprint of the British in this region.

What criticisms did the Chinese have about Britain?

The primary motive of British imperialism in China in the nineteenth century was economic. There was a high demand for Chinese tea, silk and porcelain in the British market. However, Britain did not possess sufficient silver to trade with the Qing Empire. Thus, a system of barter based on Indian opium was created to bridge this problem of payment. The subsequent exponential increase of opium in China between 1790 and 1832 brought about a generation of addicts and social instability. Clashes between the Qing government and British merchants ultimately escalated into the infamous Opium Wars. As a result, the British were given the island of Hong Kong and trading rights in the ports of Canton and Shanghai. Although British imperialism never politically took hold in mainland China, as it did in India or Africa, its cultural and political legacy is still evident today. Honk Kong remains a significant center of global finance and its government still functioned in much of the same ways as it did under British colonialism. Furthermore, the language of English and British culture highly impacted the society of Hong Kong and Southern China for over a century. This Research Guide is divided into four main components. The first section is devoted to the definitions and qualifications of imperialism. This part mainly consists of print sources that focus on the political, economic and social mechanisms of imperialism. It provides scholarly perspectives and criticisms regarding its causes and effects. The second section consists of both print and interactive sources. This section focuses on the topic of British Imperialism in China from a British perspective. The sources include various political justifications and financial factors that influence Britain's diplomatic decisions and imperialist tactics. The third section presents the Chinese perspective. The sources in this section explain the development of Chinese nationalism and the intricacies of international relations in the Qing court. The final section deals with the legacy of British imperialism in Hong Kong and southern China. The sources here examine the cultural and political footprint of the British in this region.

What did the Sepoy rebellion cause? What stereotypes did the British re-inforce?

The sepoys, a generic term used for native Indian soldiers of the Bengal Army derived from the Persian word meaning "infantry soldier", had their own list of grievances against the British East Indian Company (BEIC) administration, caused mainly by the ethnic gulf between the European officers and their Indian troops. The spark that led to a mutiny in several sepoy companies was the issue of new gunpowder cartridges for the Enfield rifle in February, 1857. A rumour was spread that the cartridges were made from cow and pig fat. Loading the Enfield required tearing open the greased cartridge with one's teeth. This would have insulted both Hindu and Muslim religious practices; cows were considered holy by Hindus while pigs were considered unclean by Muslims. Underlying grievances over British taxation and recent land annexations by the BEIC were ignited by the sepoy mutineers and within weeks dozens of units of the Indian army joined peasant armies in widespread rebellion. The old aristocracy, both Muslim and Hindu, who were seeing their power steadily eroded by the East India Company, also rebelled against British rule. The events of 1857 loom large in Indian History. ... The Revolt of 1857 led to a re-organization of the Indian army and this article highlights some of the socio-economic and cultural impacts of this re-organization. The soldier is also a product of his socio-economic, cultural and political landscape. Indian Mutiny. Indian Mutiny, also called Sepoy Mutiny, widespread but unsuccessful rebellion against British rule in India in 1857-58. Begun in Meerut by Indian troops (sepoys) in the service of the British East India Company, it spread to Delhi, Agra, Kanpur, and Lucknow. By the year 1857 the British had established complete political control of India. ... The mutiny of the Sepoy (= native troops in the British army) began on May 10, 1857, when Indian soldiers who had been placed in irons for refusing to accept new cartridges were rescued by their comrades.

Social Darwinism

The theory that individuals, groups, and peoples are subject to the same Darwinian laws of natural selection as plants and animals. Now largely discredited, social Darwinism was advocated by Herbert Spencer and others in the late 19th and early 20th centuries and was used to justify political conservatism, imperialism, and racism and to discourage intervention and reform.

What is Social Darwinism?

The theory that individuals, groups, and peoples are subject to the same Darwinian laws of natural selection as plants and animals. Now largely discredited, social Darwinism was advocated by Herbert Spencer and others in the late 19th and early 20th centuries and was used to justify political conservatism, imperialism, and racism and to discourage intervention and reform. Social Darwinism is the (mis)application of and (over) extension of Darwin's theory to explain human society. Social Darwinism uses Darwinian ideas like "evolution", "fitness", "struggle for survival" to "explain" and "justify" human inequalities in wealth and power. Applying concept of survival of the fittest to groups of people in society and using survival of the fittest to justify mistreatment of people. "Survival of the fittest" applied to society and groups of people. Social Darwinism is the theory of survival of the fittest that justifies social equality. A relevant example of social Darwinism is Nazism because it determined who was worthy enough of even surviving. - Social Darwinism is the (mis)application of and (over) extension of Darwin's theory to explain human society. - Social Darwinism uses Darwinian ideas like "evolution", "fitness", "struggle for survival" to "explain" and "justify" human inequalities in wealth and power. Types of Social Darwinism: - Individualistic Social Darwinism - Collectivistic Social Darwinism - Individualistic social Darwinism claims that human society, like nature, is involved in a constant "struggle for survival" between individuals in society. - The individuals may be businessmen competing ("struggling") against other business men for profits and economic "survival". - The individuals may also be individual workers seeking to "survive" by competing ("struggling") to get hired, earning wages, promotions, etc...

Why did Social Darwinism emerge during the Industrial Revolution?

Theory and Definition: The term Social Darwinism encompasses the theories and ideology inspired by Darwinism and associated with other evolutionary theories in the realms of politics, sociology and economics. The theory of Social Darwinism held that the weak were diminished as the strong grew in power. Adherents of Social Darwinism believed that social progress resulted from conflicts in which the fittest, or best adapted, individuals (or societies) would triumph. Social Darwinism gave rise to the slogan "survival of the fittest." The industrial revolution, on the surface, may seem like an advancement only in technology, and industry. However, there was also a lot of social change as well. People were left to survive on their own as society switched over to a capitalist's way of life. This created a survival of the fittes situation among the people of a city. There was competiton for jobs, and resouces. The people that earned the most amount of money, held a steady job, and provided for their family in a sufficient manner were deemed as the fittest. They had the most social, and economic advantages. While people left to support themselves complete factories switched from specialized workers to asembly lines to create efficiency. Because of this many people were forced to work 12- hour days far from home. This made it harder to support your family. Also, most workers were not insured if they got injured while working. If they quit they lost their jobs, so many were forced to work injured. The people that were successful often didn't work nearly as hard. However, because of their success they were socially the fittest. This gave them strong influences that brought them lots of power. Social Darwinism was a sociological theory popular in late nineteenth-century Europe and the United States. It merged Charles Darwin's theory of natural selection and Herbert Spencer's sociological theories to justify imperialism, racism, and laissez-faire (i.e. conservative) social and economic policies. Social Darwinists argued that individuals and groups, just like plants and animals, competed with one another for success in life. They used this assertion to justify the status quo by claiming that the individuals or groups of individuals at the top of social, economic, or political hierarchies belonged there, as they had competed against others and had proven themselves best adapted. Any social or political intervention that weakened the existing hierarchy, they argued, would undermine the natural order. Social Darwinism, the theory that human groups and races are subject to the same laws of natural selection as Charles Darwin had perceived in plants and animals in nature. According to the theory, which was popular in the late 19th and early 20th centuries, the weak were diminished and their cultures delimited while the strong grew in power and in cultural influence over the weak. Social Darwinists held that the life of humans in society was a struggle for existence ruled by "survival of the fittest," a phrase proposed by the British philosopher and scientist Herbert Spencer.

How unification of Germany fuels Empire

Whereas Camillo di Cavour directed Italian unification, a Junker (the Prussian name for an aristocratic landowner from old Prussia in the east) named Otto von Bismarck pushed German unification through "blood and iron" and skillful understanding of realpolitik. As the map of central Europe stood in 1850, Prussia competed with Austria for dominance over a series of small principalities fiercely keen on maintaining their independence and distinctive characteristics. Prussia proper stretched from modern-day Lithuania to central Germany. Prussia also controlled the German lands around the Rhine River in the west. In between, from Denmark to Switzerland, lay small provinces that Bismarck needed to incorporate under the Prussian crown to create a viable German Empire. In 1862, Bismarck reorganized the Prussian army and improved training in preparation for war. In 1864, he constructed an alliance with Austria to fight Denmark over Denmark's southern provinces of Schleiswig and Holstein. Prussia received Schleiswig while Austria administered Holstein. That situation, however, could not stand for long, as Austrian Holstein was now surrounded by Prussian lands. Bismarck provoked a conflict with Austria over an unrelated border dispute and in the subsequent Seven Weeks' War--named for its brevity--Prussia crushed the collapsing Austrian army. The peace settlement transferred Holstein to Prussia and forced Austria to officially remove itself from all German affairs. With Austria out of Bismarck's way, his next obstacle was the skepticism of the southern provinces. Overwhelmingly Catholic and anti-militaristic, the southern provinces doubted Prussia's commitment to a united Germany of all provinces. Prussia's Protestantism and historic militarism made the gulf between north and south quite serious. Therefore, Bismarck turned to realpolitik to unite the Germanic provinces by constructing a war against a common enemy. In 1870, Bismarck forged a note from the French ambassador, implying that the ambassador had insulted the Prussian king. After he leaked this letter to both populations, the people of France and Prussia, roused by nationalist sentiment, rose up in favor of war. As Bismarck hoped, the southern provinces rallied to Prussia's side without any hesitation. In July 1870, France declared war on Prussia. Within a matter of weeks of fighting in Alsace-Lorraine, France lost this Franco-Prussian War. Alsace-Lorraine was transferred to Germany in the peace settlement, allowing Prussia to declare the German Empire, or Second Reich, on January 21, 1871.

Boxer Rebellion

Who began the Boxer Rebellion? A Chinese secret organization called the Society of the Righteous and Harmonious Fists led to the uprising in Northern China. What happened during the Boxer Rebellion? The Society of Righteous and Harmonious Fists started with attacking foreigners and Chinese Christians. Later, the [Boxer] movement moved to Beijing where Chinese Christians & Missionaries were killed, they destroyed churches and railroad stations.On June 20, 1900, they began attacking the foreign legation district. The next day, Qing Empress announced a war on foreign nations in China. When did the Boxer Rebellion start/end? Initiated in 1900 and concluded in 1901. Where did the Boxer Rebellion take place? Began in Northern China in the Shantung province. Why did the Boxer Rebellion arise? Started with the invasion of Westerners & their Christian Missionaries and the instability in the Qing dynasty. How did the Boxer Rebellion end? Ended with the signing of the Boxer Protocol which states that the barriers that protect Beijing will be destroyed, Boxer and Chinese government officials were dismissed, and foreign legations had the right to assign troops in Beijing for defense.

Contrast direct rule vs. Indirect rule?

Who had indirect rule? Britain Who had direct rule? France, Germany, Belgium, Portugal Describe the relationship with local governments in indirect rule. Worked with local people (kings, chiefs, councils) as representatives of local autonomy Describe the relationship with local governments in direct rule Destroyed with military force due to local resistance Ultimate control in indirect rule Britain had the final say Ultimate control in direct rule ruled directly from home country and made the colony a part of France, almost like a province Economic goal in indirect rule make money-let independent companies make profits Economic goal in direct rule make money-most businesses had direct ties to government Social goal in indirect rule stressed differences with Africans, separated the races/cultures Social goal in direct rule made Africans accept the imposition of European culture-assimilation and paternalism Religious goal in indirect rule brought Protestant Christianity and educated young in new faith Religious goal in direct rule allowed local religions but pushed Catholicism as being more European New ideas in indirect rule taxes, forced public labor, control population mobility New ideas in direct rule taxes, warfare, language, customs, government What was education like in indirect rule? educate the masses enough to make them better workers, educate some a lot but don't give them positions of power What was education like in direct rule? educate the few elite a lot to make them Europeans What was the result of indirect rule? Corrupted existing social and government structures, introduced British legal system, made Africans more British What was the result of direct rule? close ties between mother country and peoples of the colonies, destroyed local African cultures Evaluation of indirect rule - very successful in some nations(Nigeria) but big problems in others (Kenya) Evaluation of direct rule - very successful for the French but less so for the other European nations

Unification of Germany

With all this territory now in Prussian control and its borders secure, Bismarck declared the German Empire in 1871, crowning the King of Prussia, Wilhelm I, as Kaiser of Germany. The ceremony took place in Versailles, the traditional seat of French power, further humiliating France. Whereas Camillo di Cavour directed Italian unification, a Junker (the Prussian name for an aristocratic landowner from old Prussia in the east) named Otto von Bismarck pushed German unification through "blood and iron" and skillful understanding of realpolitik. The Franco Prussian war lead to the unification of Germany by the Austro-Prussian War joined together the north German states. After Prussia won the Franco-Prussian War, the peace treaty declared the unification of Germany.On 18th January, 1871, after the defeat of France in the Franco-Prussian War, the various princes of the German states united and proclaimed Wilhelm of Prussia as emperor of the German Empire with Bismarck as the Chancellor. The unification of Germany into a politically and administratively integrated nation state officially occurred on 18 January 1871, in the Hall of Mirrors at the Palace of Versailles in France. The humiliating defeat of Louis Napoleon's Second Empire of France is made complete on May 10, 1871, when the Treaty of Frankfurt am Main is signed, ending the Franco-Prussian War and marking the decisive entry of a newly unified German state on the stage of European power politics, so long dominated by the great empires of England and France.

Division of Labor

Workers were able to specialize and perform specific tasks. Product moves from worker to worker, as each one performs a step in the manufacturing process. With this division of labor, workers can make many items quickly. • Factory owners divided the manufacturing process into steps • Increased mass production • The use of machines in many of the steps helped the workers produce more in less time • Lowered cost of production • Made more profit for the owners • The manufacturing process was divided into steps • An unskilled laborer was hired and assigned a step • This increased production • Machines helped workers produce more in shorter time frame which lowered cost of production


Related study sets

Unit 4 Breast Cancer, Leukemia and Cellulitis

View Set

Principles of Macroeconomics: Chapter 3 Study Guide

View Set

Module 14 Inflation: An Overview

View Set

14.2 Anxiety Disorders, OCD, and PTSD

View Set

Breast Cancer ch.51 Med Surg (EXAM#2)

View Set

chapter 11 cervical and thoracic spinal conditions

View Set

Fundamentals Nursing Chapter 9 Care Coordination and Continuity in Health Care Settings and the Community

View Set